StudyWithMeenakshi

Search Your Topic Here

Sunday, July 08, 2018

BANKING MATERIAL: MCQ COLLECTION

Important MCQs

1. Many times we read in newspapers that several companies are adopting the FCCBs route to

raise capital. What is the full form of FCCBs?

(1) Foreign Currency Convertible Bonds

(2) Foreign Convertible Credit Bonds

(3) Financial Consortium and Credit Bureau

(4) Future Credit and Currency Bureau

(5) None of these

Ans: 1



2. Many a times, we read in newspapers that some big banks have revised their lending rates to

make them dearer or cheaper. Though the decision to raise the lending rates is always in the hands

of the banks normally they announce this decision of theirs:

(1) Immediately after the Union Budget is presented in the Lok Sabha every year.

(2) when the RBI makes changes in its policy rates.

(3) When the Monetary and Credit Policy of the RBI is reviewed periodically.

(1) Only 1 (2) Only 2

(3) Only 3 (4) Only 2 and 3

(5) All 1, 2 and 3

Ans: 3

3. In order to develop infrastructure in the country at a faster pace, the Govt. of India has fixed some

targets. In order to achieve this target, at present how many kilometers of highway is required to be

constructed per day?

(1) 5 km (2) 10 km

(3) 15 km (4) 20 km

(5) All 1, 2 and 3

Ans: 4

4. The Govt. of India is planning to bring a Second Green Revolution. This will be launched

specifically for which of the following parts of the country?

(1) North East and Eastern Regions

(2) Central India

(3) Jammu & Kashmir

(4) Tamil Nadu, Kerala and Andhra Pradesh

(5) None of these

Ans: 1

5. Inflation in India is measured on which of the following indexes/indicators?

(1) Cost of Living Index (COLI)

(2) Consumer Price Index (CPI)

(3) Gross Domestic Product (GDP)

(4) Wholesale Price Index (WPI)

(5) None of these

Ans: 4

6. Which of the following is a function of the agent bank relating to flow of money in the process of

loan syndication?

(1) Receiving notices relating to cancellation of any part of the loan

(2) Calling of loans in the event of default

(3) Receiving the fee from the borrower and distributing among the participating banks

(4) Receiving notices relating to transfer of banks

(5) None of these

Ans: 3

7. Which of the following is a leading electronic payment technology firm?

(1) Visa (2) Max

(3) BSE (4) Sensex

(5) SWAP

Ans: 1

8. CRR funds are kept by the Banks in…………

(1) Cash in hand at branches

(2) Balance with other banks

(3) Balance in a special Account with RBI

(4) Funds in the currency chest

(5) None

Ans: 3

9. In the list of nationalized banks, which bank was merged with Punjab National Bank in 1993?

(1) Bank of India

(2) Bank of Baroda

(3) Andhra Bank

(4) New Bank of India

(5) Central Bank of India

Ans: 4

10. The Shipping guarantee is a?

(1) Type of a letter of credit

(2) Guarantee issued by the ship captain to the purchaser

(3) Guarantee issued to the borrower towards the loan granted by the shipping company

(4) Deferred payment guarantee issued by a banker at the request of the consignee when the

documents are not received and goods are received, for facilitating the delivery of goods.

(5) None of these

Ans: 4

11. Who said the nationalization of banks is the ‘masterstroke of political sagacity?

(1) Indira Gandhi

(2) Raj Narayan

(3) Jayaprakash Narayan

(4) Jawahar Lal Nehru

(5) Atal Bihari Vajpayee

Ans: 3

12. Regional Rural Banks have been set up with the basic objectives of:

(1) Providing credit to semi urban & urban population

(2) Providing deposits facilities to farmers

(3) Providing credit and deposit facilities to rural areas

(4) Providing credit, deposit and other banking facilities to people in rural areas

(5) None of these

Ans: 4

13. Global Market means market of:

(1) USA

(2) China

(3) OPEC members

(4) European union countries

(5) All major countries and their alliances

Ans: 5

14. Which of the following is not a financial intermediary in the financial markets?

(1) Pension funds

(2) Credit unions

(3) Life Insurance companies

(4) Small scale service institute

(5) None of these

Ans: 4

15. The interest on recurring deposit is paid on the basis of?

(1) Simple interest calculated monthly basis

(2) Simple interest on monthly products basis

(3) Quarterly compounding

(4) Interest calculated on daily products basis

(5) None of these

Ans: 3

16. Which of the following is not one of the RBI directives on clean note policy?

(1) Currency note packets are not to be stapled and secured with paper bonds

(2) Soiled notes are to be stapled before they are remitted to Currency Chest

(3) Water mark window of bank notes shall not contain any writings

(4) Currency notes are to be sorted into Issuable and non-issuable notes

(5) None of these

Ans: 2

17. When bank accept a fixed sum of money from an individual for a definite term and pay on

maturity with interest, the deposit is known as:

(1) Term Deposit (2) Demand Deposit

(3) Bond (4) Mortgage

(5) Advance

Ans: 1

18. Bad debts mean……………?

(1) amounts owed to a company that are going to be paid

(2) amounts owed to a company that are not going to be paid

(3) amounts owed to a company that are going to be paid within 2 years

(4) amounts owned to a company that are not going to be paid within 8 years

(5) None of these

Ans: 2

19. In which part of India, many banks are established, which is called the Cradle of Indian Banking

System?

(1) Northern Assam

(2) Southern Tamilnadu

(3) Eastern Rajasthan

(4) Southern Karnataka

(5) Western Maharastra

Ans: 4

20. Scheduled banks are those?

(1) Included in the 2nd schedule of the Banking Regulation Act 1949

(2) Included in the 2nd schedule of the Companies Act 1956

(3) Included in the 2nd schedule of the Reserve Bank of India Act – 1934

(4) Bank Nationalization Act – 1969

(5) None of these

Ans: 3

21. Which of the following is not an imperfect Note?

(1) Washed note (2) Bleached note

(3) Mutilated note (4) Oiled note

(5) None of these

Ans: 3

22. The relationship between RBI and the Bank maintaining the currency chest is that of?

(1) Debtor and Creditor

(2) Principal and Agent

(3) Bailor and Bailee

(4) Creditor and Debtor

(5) None of these

Ans: 2

23. Which of the following committees are related to ‘Micro financing?

(1) Nayak Committee

(2) Tandon Committee

(3) Vyas Committee

(4) Narasimham Committee

(5) None of these

Ans: 3

24. Which of the following organizations /agencies works solely to monitor and arrange flow of

agriculture credit in India?

(1) NABARD (2) SIDBI

(3) RBI (4) SEBI

(5) None of these

Ans: 1

25. Which is incorrect with regard to powers of RBI:

(1) Holds cash reserve banks

(2) Controls banking system through licensing

(3) Supervises Banking system through inspection

(4) All of the above are correct

(5) All of the above are incorrect

Ans: 4

26. India’s first indigenous payment gateway is ……….?

(1) India Expenditure (2) India Payment

(3) India Challan (4) India Pay

(5) India Rupee

Ans: 4

27. ………...opened a Lockless branch at Shani Shingnapur (Maharastra) respecting the local

Customs?

(1) State Bank of India

(2) Bank of Maharastra

(3) Union Bank of India

(4) ICICI

(5) UCO Bank

Ans: 5

28. Security printing press was established in 1982 at?

(1) Kolkata (2) New Delhi

(3) Bombay (4) Hyderabad

(5) None of these

Ans: 4

29. When the customer withdraws cash from ATM, the banker and customer relationship is?

(1) Debtor and Creditor

(2) Creditor and Debtor

(3) Lesser and Lessee

(4) Agent and principal

(5) None of these

Ans: 1

30. In India, forex rates are determined by:

(1) Importers

(2) Exporters

(3) RBI

(4) Ministry of commerce

(5) Demand and supply forces

Ans: 5

31. The Government of India has recently given its approval to expand number of seats available

weekly on flights between India and United Arab Emirates (UAE) from the current 13,000 seats

to………?

(1) 25000 (2) 30000

(3) 20000 (4) 50000

(5) None of these

Ans: 4

32. Regional Rural Banks were set up on the recommendations of:

(1) Nariman committee

(2) Narasimham committee

(3) Gadgil committee

(4) Puri committee

(5) None of these

Ans: 2

33.………….launched the process of reforms of financial system in India:

(1) Gadgil committee

(2) Nariman committee

(3) Narasimham committee

(4) Khanna Committee

(5) None of these

Ans: 3

34. Which of the following can be categories as a Merchant Banking Service?

(1) Consultancy on finance to a company

(2) Advance in capital structure

(3) Managing mergers

(4) Helping in finalizing take over

(5) All the above

Ans: 5

35. The instrument which provides proprietary interest in company and which entitles a person to

dividend on profits, is called?

(1) Bond (2) Share

(3) Debenture (4) Warrant

(5) Any of the above

Ans: 2

36. Which of the following provides proper definition of a Mutual Funds?

(1) It is an association of members of public

(2) These members want to ‘invest in financial instrument

(3) They want to invest in financial instrument assets of business sector

(4) All the above

(5) None of these

Ans: 4

37. Service Area Approach is associated with?

(1) Industrial finance

(2) Microfinance

(3) Rural finance

(4) Consortium finance

(5) None of these

Ans: 3

38. Which of the following resources cannot be Securitized?

(1) Credit balances outstanding in cash credit accounts

(2) Credit Card receivables

(3) Hire purchase receivable

(4) Mortgage in lieu of future payments

(5) None of these

Ans: 1

39. Which of the following is a type of anticipatory letter of credit?

(1) Green clause letter of credit

(2) Yellow clause letter of credit

(3) Back to back letters of credit

(4) Revolving letter of credit

(5) None of these

Ans: 1

40. Interest on Government securities on fixed basis is known as:

(1) Bank rate

(2) Coupon

(3) Prime Lending Rate

(4) Bench mark PLR

(5) None of these

Ans: 2

41. ………….was not nationalized in 1969:

(1) Punjab National Bank

(2) Oriental Bank of Commerce

(3) Bank of Baroda

(4) Union Bank of India

(5) None of these

Ans: 2

42. In the first round 14 major banks with a liability base of……………were nationalized:

(1) 10 crore or more

(2) 20 crore or more

(3) 50 crore or more

(4) 100 crore or more

(5) None of these

Ans: 3

43. ……………do not attract TDS:

(1) Fixed deposits

(2) Reinvestment deposits

(3) NRO SB

(4) NRO FDR

(5) Recurring deposits

Ans: 5

44. SLR is maintained as a percentage of:

(1) Time Liabilities

(2) Demand liabilities

(3) Time and Demand liabilities

(4) Gross time and demand liabilities

(5) Net demand and time Liabilities

Ans: 5

45. The credit policy laid down by the top management of a bank does not deal with?

(1) Credit risk management

(2) Pricing of the credit products

(3) Appraisal of time and demand deposits

(4) Documentation standards

(5) None of these

Ans: 3

46. Which of the following is an income for a Bank?

(1) Depreciation on bank’s property

(2) Interest on inter bank borrowings

(3) Profit on revaluation of investments

(4) Provision made on account of write-offs

(5) None of these

Ans: 3

47. Which of the following is not the function of commercial banks?

(1) Providing transaction services

(2) Intermediation in financial services

(3) Providing transformation services

(4) Regulating the issue of bank notes

(5) None of these

Ans: 4

48. Which of the following is one of the three pillars of new Basel Capital Award?

(1) Market competitions

(2) Market rapport

(3) Market discipline

(4) International market relations

(5) None of these

Ans: 3

49. Which of the following is not the name of a bank functioning in India?

(1) Central Bank of India

(2) UCO Bank

(3) Aegon Religare

(4) Dena Bank

(5) ICICI Bank

Ans: 3

50. Which of the following is not augmented feature of a credit card?

(1) Personal accident insurance

(2) Cash withdrawals

(3) Add-on facility

(4) Issue of deferred guarantee

(5) None of these

Ans: 4

51. Which of the following is the disadvantage of going for public issue?

(1) Liquidity to existing shares

(2) Increase in visibility and reputation to the company

(3) Better pricing and placement with new investors

(4) Need to make continuous disclosures

(5) None of these

Ans: 4

52. Which of the following is not a criterion to select the Investment Bankers?

(1) No professional memberships or incorporations are required

(2) General reputation in the market

(3) Good rapport with market intermediaries

(4) Distribution net work of the organization

(5) None of these

Ans: 1

53. Which of the following is not an asset of a Bank?

(1) Notes and small coins

(2) Overdue recurring deposits

(3) Short term loans

(4) Staff advances

(5) None of these

Ans: 2

54. The banking company has restriction to sanction loan to?

(1) Directors of the bank

(2) Staff working in the bank

(3) Students going abroad

(4) Professionals

(5) None of these

Ans: 1

55. Which of the following is a non-depository Institution?

(1) Credit unions

(2) Commercial banks

(3) Mutual funds

(4) Regional rural banks

(5) None of these

Ans: 3

56. The credit policy of a bank does/not deal With?

(1) Credit risk management

(2) Documentation standards

(3) Review and renewal of advances

(4) Outstanding balances in deposit accounts

(5) None of these

Ans: 4

57. Issuing credit cards is a component of?

(1) Corporate banking

(2) Rural banking

(3) Retail banking

(4) Micro finance

(5) None of these

Ans: 3

58. The base rate is set by

(1) Individual banks

(2) RBI

(3) Government of India

(4) RBI in consultation with Government

(5) None of these

Ans: 1

59. State Bank of India’s new floating rate of deposit is directly linked to?

(1) Inflation Rate

(2) BPLR

(3) Base Rate

(4) Bank Rate

(5) None of these

Ans: 3

60. Which of the following businesses is prohibited as per Sec 8 of B.R. Act 1949?

(1) Executing the trusts

(2) Leasing

(3) Retail trading

(4) Underwriting

(5) None of these

Ans: 3

61. An increase in CRR by RBI leads to:

(1) Decrease in deposit

(2) Increase in deposit

(3) Increase in lendable resources

(4) Decrease in lendable resources

(5) None of these

Ans: 4

62. When RBI reduce the CRR it results into:

(1) Decrease in deposit

(2) Increase in deposit

(3) Increase in lendable resources

(4) Decrease in lendable resources

(5) None

Ans: 3

63. What is meant by development Oriented Banking?

(1) Infrastructure financing

(2) Extending loans to States, which are undevelopment

(3) Taking up the task of development of the economy by providing support to under privileged

sections of the society

(4) All the above

(5) None of these

Ans: 3

64. Principal functions of SIDBI:

(1) Promotion of SSI

(2) Finance of SSI

(3) Development of SSI

(4) All the above

(5) None of these

Ans: 4

65. ……….heads and conducts the affairs of RBI:

(1) Central Board

(2) Local Board

(3) Regional board

(4) All of the above

(5) None of these

Ans: 1

66. RBI Provides………..for meeting day-to-day receipt and expenditure mismatch to both Central

and State Governments.

(1) treasury bills

(2) Ways and Means advance

(3) date and securities

(4) All the above

(5) None of these

Ans: 2

67. RBI known as lender of last resort because:

(1) It has to meet the credit need of citizens to whom no one else is willing to lend

(2) Banks lend to go to RBI as a last resort

(3) It comes to help banks in times of crisis

(4) All of the above

(5) None of these

Ans: 3

68. These days RBI uses Selective credit control measure rather infrequently because of:

(1) Deregulation of functions

(2) Autonomy given to banks

(3) Comfortable liquidity

(4) Reasonable inflation level

(5) All the above

Ans: 5

69. The following is classified as a public sector Bank?

(1) ICICI Bank (2) IDBI Bank Ltd

(3) Axis Bank (4) Local area bank

(5) None of these

Ans: 2

70. The banker-customer relationship in credit card payment is?

(1) Creditor-debtor (2) Debtor-creditor

(3) Agent-principal (4) Principal-agent

(5) None of these

Ans: 2

71. The availability of cash and other cash-like marketable instruments that are useful in purchase

and investments are commonly known as:

(1) Cash crunch (2) Liquidity

(3) Credit (4) Marketability

(5) None of these

Ans: 2

72. As per newspaper reports, India’s trade gap has been showing a negative growth for the last

few months. “Trade Gap” means what?

(1) Gap between total GDP and total consumption

(2) Gap between total imports and total exports

(3) Gap between available liquidity and expected demand in next five months

(4) Gap between budgeted revenue collection and actual collection of the same

(5) None of these

Ans: 2

73. RBI Provides……………for meeting day-today receipt and expenditure mismatch to both

Central and State Governments.

(1) treasury bills

(2) Ways and Means advance

(3) date and securities

(4) All the above

(5) None of these

Ans: 2

74. Who has been appointed as the new President of the Asian Development Bank (ADB)?

(1) Takehiko Nakao

(2) Kikuo Iwata

(3) Haruhiko Kurado

(4) Hiroshi Nakaso

(5) None of these

Ans: 1

75. Which of the following instruments of credit control adopted by the Reserve Bank of India (RBI)

does not fall within ‘general’ or ‘quantitative’ methods of credit control?

(1) Stipulation of certain minimum margin in respect of advance against specified commodities

(2) Open Market operation

(3) Bank rate

(4) Variable reserve requirement

(5) None of these

Ans: 1

76. The term ‘BSR’ refers to:

(1) Bank’s Selling Rate

(2) Basic Statistical Returns

(3) Annual returns submitted by banks to RBI in respect of priority sector advances

(4) Quarterly statement of advances to agriculture

(5) None of the above

Ans: 2

77. Participatory Notes (PNs) are associated with which one of the following?

(1) Consolidated Fund of India

(2) Foreign Direct Investors

(3) Foreign Institutional Investors

(4) United Nations Development Programme

(5) None of these

Ans: 3

78. Devaluation means:

(1) To reduce the value of home currency

(2) To appreciate the value of the home currency

(3) To issue new currency in place of old currency

(4) To lower the prices of goods for export

(5) None of these

Ans: 1

79. Increase in net RBI credit for central government represents:

(1) Budgetary deficit (2) Revenue deficit

(3) Fiscal deficit (4) Monetised deficit

(5) None of these

Ans: 4

80. A country is said to be in a debt trap if:

(1) It has to borrow to make interest payments on outstanding loans

(2) It has to borrow to make interest payments on standing loans

(3) It has been refused loans or aid by creditors abroad

(4) The World Bank charges a very high rate of interest on outstanding as well as new loans

(5) None of these

Ans: 2

81. Bank Rate implies the rate of interest:

(1) Paid by the Reserve Bank of India on the deposits of commercial banks

(2) Charged by banks on loans and advances

(3) Payable on bonds

(4) At which the Reserve bank of India discounts the Bills of Exchange

(5) None of these

Ans: 4

82. Which among the following committee is set up for giving clear definitions to FDI and FII with an

aim to remove ambiguity over the two types of foreign investments?

(1) Gopinath committee

(2) Subir Gokran committee

(3) Arvind Mayaram committee

(4) Sushma Nath committee

(5) None of these

Ans: 3

83. As per the new final guidelines released by the Reserve Bank of India (RBI) for issuing new

banking licenses norms to allow private and the public sector as well as non-banking financial

companies to enter the fray. Which of the following statements is Correct in this regard?

(1) The final guidelines sets 49 percent cap on foreign holding in new banks and minimum paid-up

equity capital is Rs. 1000 crore.

(2) New banks will have to set up 25 per cent of its branches in unbanked rural areas.

(3) The paid-up equity capital should be

Rs. 500 crore and they will have to get listed within three years of operations.

(1) Only A & B (2) Only B and C

(3) Only A & C (4) All A, B and C

(5) None of these

Ans: 2

84. The rate of interest, banks charge to its main/major and prime; customers is popularly called as:

(1) Prime Lending Rate (2) Repo Rate

(3) Cost of Fund (4) Risk Premium

(5) Reverse Repo Rate

Ans: 1

85. Currency Swap is an instrument to manage?

(1) Interest rate risk

(2) Currency risk

(3) Currency and interest rate risk

(4) Cash flows in different currencies

(5) All of the above

Ans: 4

86. When a bank dishonours a cheque……….?

(1) It is called withdrawing of the cheque

(2) It is called settlement of the cheque

(3) It is called nullifying of the cheque

(4) It is called return of the cheque unpaid

(5) It is called truncating of the cheque

Ans: 4

87. Loans of very small amounts given to low income groups is called:

(1) Micro Credit (2) Cash Credit

(3) Simple Overdraft (4) Rural Credit

(5) No Frills Loan

Ans: 1

88. Under provisions of which one of the following Acts, the Reserve Bank of India (RBI) issues

directives to the Banks in India?

(1) Banking Regulation Act

(2) Essential Commodities Act

(3) RBI Regulation Act

(4) RBI an Banking Regulation Act

(5) None of these

Ans: 4

89. In banking terms, BCSBI stands for which of the following?

(1) Banking Credit and Securities Board of India

(2) Banking Codes and Standards Board of India

(3) Banking Consumers and State Board of India

(4) Banking Commerce and Secretarial Board of India

(5) Banking Communications and Systems Board of India

Ans: 2

90. When we deposit a cheque issued in our name in the bank, the bank always checks if the

cheque has been crossed or not? Why is this done?

(1) It is a process by which the person who has issued the cheque comes know whether the cheque

is enchased or not

(2) It ensures that the money is deposited only in the account of the person in whose name the

cheque has been drawn

(3) The bank insists on it only when the party wants the payment immediately and that too in cash

only

(4) This is the instruction of RBI that all the cheques of the amount of Rs. 10,000 should be accepted

only if they are crossed

(5) None of these

Ans: 2

91. When the loan is guaranteed for purchase of white goods it is called?

(1) White goods loan

(2) Consumer durable loan

(3) Business loan

(4) Consumption loan

(5) Proprietary loan

Ans: 2

92. Who among the following is appointed to head a 10-member global task force to prepare a

Knowledge management system for promoting global cooperation in anticorruption measures and

keep a check on balance money?

(1) Kuldeep Khoda

(2) Pradeep Kumar

(3) Vinod Mehta

(4) Rajesh Chauhan

(5) None of these

Ans: 2

93. Which among the following is not directly controlled by RBI?

(1) Bank Rate

(2) Base Rate

(3) Repo Rate

(4) CRR

(5) SLR

Ans: 3

94. A loan bearing low rate of interest is known as:

(1) Hard loan (2) Soft loan

(3) Capital loan (4) Real loan

(5) None of these

Ans: 2

95. World Bank Group announced the appointment of an Independent panel of experts to conduct a

review of it’s Doing Business report. Who among the following from India has been appointed as one

of the ten International experts to this panel?

(1) Dr. K. Kasturirangan

(2) B.K. Chaturvedi

(3) Narendra Jadhav

(4) Arun Maira

(5) None of these

Ans: 4

96. Which among the following is the High-level committee that went into the Commonwealth

Games related projects, has suggested changes in the functioning and structure of top audit and

vigilance bodies like the Comptroller and Auditor General making it a three-member body?

(1) U.C. Banerjee Committee

(2) Shunglu Committee

(3) Palekar Committee

(4) A Ghosh Committee

(5) None of these

Ans: 2

97. Sub-Prime lending refers to:

(1) lending to the customers who are most value for the banks

(2) lending to the customers who visit bank for the very first time

(3) lending to the people with less than standard credit status

(4) lending to the people who live a substandard life

(5) None of these

Ans: 3

98. Which of the following is the deadline set by RBI for Indian Banks to complete their conformation

to the Basel-III norms?

(1) March 2020

(2) March 2016

(3) March 2018

(4) March 2014

(5) None of these

Ans: 3

99. CAPART is associated with which of the Following?

(1) Energy Sector

(2) Rural Development

(3) Infrastructure

(4) Foreign trade

(5) None of these

Ans: 2

100. To which of the following banks of Russia, the Reserve Bank of India (RBI) has granted

permission to open its branch in New Delhi?

(1) Active Bank

(2) Bank BFT

(3) Inkas Bank

(4) Gazprom Bank

(5) MDM Bank

Ans: 3

101. Certificates of Deposits have a minimum value of:

(1) Rs. 1 lakh

(2) Rs. 10 lakh

(3) Rs. 25 lakh

(4) Rs. 1 crore

(5) None of these

Ans: 1

102. Commercial paper can be issued:

(1) By all corporate

(2) By all corporate with net worth of t least Rs. 10 crore

(3) By all corporate with net worth of at least Rs. 5 crore

(4) Can be issued only by banks

(5) None of these

Ans: 3

103. Which of the following is not true?

(1) Call money deals with overnight loans

(2) As special cases, few FIs like LIC, UTI can borrow in the call money market

(3) Call loans are made on a ‘clean basis’

(4) Is a part of organized money market

(5) None of these

Ans: 2

104. Planning Commission has set up a high level expert committee to suggest measures for

efficient management of public – expenditure under the chairmanship of…………

(1) Prof. Ravindra Dholakia

(2) C. Rangarajan

(3) K. Kasturirangan

(4) Nitin Desai

(5) None of these

Ans: 2

105. A bank accepts a deposit from a corporate house. The feature of the deposit are:

(a) accepted at a discounted value

(b) stamp duty is borne by the bank

(c) issued as issuance of promissory note

(d) TDS is not applicable.

Identify the deposit:

(1) Commercial paper

(2) Certificate of deposit

(3) Flexi deposit

(4) Caution deposit

(5) None of these

Ans: 2

106. National Savings Certificate matures at the end of:

(1) 6 years (2) 3 years

(3) 5 years (4) 10 years

(5) None of these

Ans: 3

107. Many a Times, we read in Financial Newspapers about a term called “Loan Syndication”. What

exactly is Loan Syndication?

(1) In Loan Syndication, more than one debtors applying for a loan to one creditor

(2) In Loan Syndication, more than one creditors coming together to provide single loan

(3) Loan Syndication, more than one creditors coming together to provide multiple loans

(4) All of the above

(5) None of these

Ans: 2

108. This committee is associated with finance to small scale industries:

(1) C.E. Kamath (2) Malegan

(3) G.S. Patel (4) Chatelier

(5) Khanna

Ans: 2

109. If the government chooses to spend money freshly printed by the RBI against government

securities, it should be kept in which among the following?

(1) Fiscal Adjustment

(2) Deficit Financing

(3) Mandatory spending

(4) Retrenchment

(5) None of these

Ans: 2

110. The newly inaugurated helpline ‘Udyami’ is to assist:

(1) Micro, Small and Medium Enterprises

(2) Large Capital industries

(3) Only Female Entrepreneurs

(4) Farmers introducing technology in framing

(5) None of these

Ans: 1

111. The Statutory Liquidity Ratio (SLR), the amount of liquid assets such as cash, precious metals

and other short-term securities are kept with:

(1) RBI

(2) Individual banks

(3) Finance Ministry

(4) A bank designated by RBI

(5) None of these

Ans: 2

112. RBI hike policy rates by 25 basis point in percentage term it is equivalent to

(1) 0.25% (2) 25%

(3) 0.25% (4) 2.5%

(5) .0025%

Ans: 3

113. Bhandari Committee is related to:

(1) Entry of private players in telecommunication sector

(2) Organisation of Railway Zones

(3) Reconstruction of RRBs

(4) Reforms in oil sector

(5) None of these

Ans: 3

114. Which among the following committee has looked into the maladies affecting the proper

functioning of the public distribution systems (PDS) in India?

(1) Suresh Tendulkar Committee

(2) Kaushik Basu Committee

(3) Wadhwa Committee

(4) Rangarajan Committee

(5) None of these

Ans: 3

115. ARCIL is an example of:

(1) A financial institution

(2) A mutual fund

(3) An asset management company set up toacquire NPAs of banks

(4) A discount and financing house

(5) None of these

Ans: 3

116. Income limit under Rajiv Gandhi Equity Savings Scheme (RGESS) will be raised from Rs.10

lakh to……….?

(1) Rs. 11 lakh (2) Rs. 13 lakh

(3) Rs. 12 lakh (4) Rs. 15 lakh

(5) None of these

Ans: 3

117. NABARD has started its Farmer’s Club Scheme which plays a major role in uplifting the Sheep

and Goat Rearing Sector and Pashmina Rearing in which of the following States?

(1) Jammu and Kashmir (2) Andhra Pradesh

(3) Maharashtra (4) Sikkim

(5) None of these

Ans: 1

118. The Prime Minister’s Economic Advisory Council (PMEAC) under the Chairmanship of C.

Rangarajan has presented the Economic review 2012-13. Which of the following highlight/s is/are

true?

A. India’s economic is expected to grow 6.4 percent in the new financial year that began on 1 April

2013

B. PMEACH pegged the WPI inflation at around 6 percent and food inflation, at 8 percent.

C. FY 13 bank credit growth at 14.1% vs 17% in the year-ago period.

D. Net FDI at 18 billion dollars and FII inflows at 24 billion dollar in FY13, expecting net FDI inflow at

24 billion dollars and FII at 18 billion dollars for FY14

(1) Only A and B (2) A and C

(3) A, B and D (4) All of above

(5) None of these

Ans: 4

119. Which of the following states has launched a digital Krishi Card which is supposed to help

farmers in making use of benefits such as subsidies and loans given by the State Government?

(1) Kerala (2) Maharashtra

(3) Goa (4) Uttar Pradesh

(5) None of these

Ans: 3

120. Which of the following states has issued Shiksha Setu Cards for Government Schools to keep

students and teachers updated about the performance of students?

(1) Andhra Pradesh (2) Haryana

(3) Bihar (4) Maharashtra

(5) None of these

Ans: 2

121. An Inter-Ministerial Group (IMG) has approved 10 percent equity sale of Coal India Limited

(CIL). At Present Union Government holds over how much percent stake in Coal India Ltd?

(1) 80% (2) 75%

(3) 90% (4) 60%

(5) None of these

Ans: 3

122. The Cabinet Committee on Economic Affairs (CCEA) has recently approved the proposal of

the Ministry of Shipping for setting up of two major ports in the country in which of the following

states?

(1) West Bengal and Andhra Pradesh

(2) Madhya Pradesh and Andhra Pradesh

(3) Maharashtra and Kerala

(4) Maharashtra and West Bengal

(5) None of these

Ans: 1

123. Supreme Court has given green signal to operationalize the Kudankulam Nuclear Power plant.

In which of the following states this plant is located?

(1) Andhra Pradesh (2) Tamil Nadu

(3) Kerala (4) Orissa

(5) None of these

Ans: 2

124. Asian Development Bank would provide about $6 billion loan to India over the next how many

years, even it is facing the challenge of raising resources?

(1) 2 (2) 4

(3) 3 (4) 6

(5) None of these

Ans: 3

125. Which of the following investigative websites alleged that at least 23 top public and private

banks and insurance companies have been doing large-scale money laundering?

(1) Cobrapost (2) Tahelka

(3) Wikileaks (4) None of above

(5) None of these

Ans: 1

126. World bank assisted Rs. 2893 crore Project for Child Nutrition Programme. What is the name

of this project?

(1) ICDS Systems Strengthening and Nutrition Improvement Project (ISSNIP)

(2) ICDS Systems and Nutrition Project (ISNP)

(3) ICDS Systems and Nutrition Improvement Project (ISNIP)

(4) ICDS Systems Strengthening and Improvement Project (ISSIP)

(5) None of these

Ans: 1

127. To harmonise and co-ordinate the skill development efforts of the government and the private

sector, the Cabinet recently approved the setting up of an autonomous body. What is the name of

this body?

(1) National Skill Development Agency

(2) National Coordination and Skill Development Agency

(3) National Agency of Coordination

(4) Skill Development Agency of India

(5) None of these

Ans: 1

128. The Government of India has reduced tax deduction at source (TDS) on interest payments for

foreign institutional investors (FIIs) on investments made by them in Government Securities (GSecs)

and rupeedenominated corporate bonds from 20% to…………..?

(1) 12% (2) 15%

(3) 10% (4) 5%

(5) None of these

Ans: 4

129. According to the World Bank’s latest India Development Update, a bi-annual report on the

Indian economy, India is expected to record how much % gross domestic product (GDP) growth in

the current fiscal 2013-14?

(1) 6% (2) 5.5%

(3) 6.1% (4) 6.2%

(5) None of these

Ans: 3

130. Which of the following stock exchanges recently launched an Islamic equity index based on the

wide-measure covering 500 companies?

(1) Bombay Stock Exchange

(2) National Stock Exchange

(3) Multi Commodity Exchange

(4) Islamic Equity Exchange

(5) None of these

Ans: 1

131. The Reserve Bank of India (RBI) has directed banks to follow the Clean Note Policy strictly and

issue clean currency notes to public. What does Clean Note Policy means?

(1) To do away the process of stapling the currency notes and to secure the note packets with paper

bands.

(2) To sort notes into re-issuable and non issuable notes and to withdraw soiled notes from

circulation in the market.

(3) To stop writing of any kind on watermark window of bank notes.

(4) All of above

(5) None of these

Ans: 4

132. The Union Government of India has hiked the monetary assistance under the Indira Awas

Yojana (IAY), the housing scheme for the poor, giving priority to the scheduled castes, tribes and

minorities from Rs. 45,000/- to Rs…………

(1) 80,000 (2) 60,000

(3) 50,000 (4) 70,000

(5) None of these

Ans: 4

133. iCED was recently inaugurated by the Vice President Hamid Ansari in Jaipur. What is the full

form of iCED?

(1) International Centre for Environmental Development

(2) International Centre for Educational Development

(3) International Centre for Environment Audit and Sustainable Development

(4) International Centre for Economy Development

(5) None of these

Ans: 3

134. The Union Government of India has recently revised the criteria for creamy layer of Other

Backward Classes (OBCs) to avail benefits of reservations in government jobs and admissions to

central educational institutions from Rs. 4.5 lakh to Rs……………?

(1) 5 lakh (2) 6 lakh

(3) 5.5 lakh (4) 7 lakh

(5) None of these

Ans: 2

135. India recently nominated hill forts of Rajasthan for UNESCO’s World Heritage Sites for 2013.

How many hill forts are Nominated?

(1) 7 (2) 8

(3) 6 (4) 5

(5) None of these

Ans: 3

136. A Memorandum of Understanding (MoU) was recently signed between National Commission

for Women (NCW) and HUDCO for improving living conditions of destitute women. What is the full

form of HUDCO?

(1) Housing and Urban Development Corporation Limited

(2) Health and Urban Development Corporation Limited

(3) None of above

(5) None of these

Ans: 1

137. The Union Government has decided to publicise best practices on implementation of

Right to Information (RTI) Act by public authorities all over the country. In which of the following

years the Centre’s flagship Right to Information Act was enacted?

(1) 2004 (2) 2003

(3) 2005 (4) 2006

(5) None of these

Ans: 3

138. Which of the following Union Territories has launched central government sponsored Rashtriya

Kisan Yojana Scheme to promote dairy activities and to enhance socioeconomic status of the

farmers?

(1) Lakshadweep (2) Delhi

(3) Puducherry (4) Daman and Diu

(5) None of these

Ans: 3

139. Federal Space Agency Roscosmos, Russia has launched the world’s only returnable satellite.

What is the name of this satellite?

(1) Bion-M1 (2) Bion-M2

(3) Sputnik-1 (4) Kosmos 2175

(5) None of these

Ans: 1

140. Mexico is the fourth largest recipient of remittances in the world, receiving 23 billion US Dollars

in 2012, according to the latest edition of the World Bank’s Migration and Development Brief. Which

of the following countries is at the top position.

(1) China (2) India

(3) Philippines (4) Egypt

(5) None of these

Ans: 2

141. Roberto Azevedo was appointed the head of the World Trade Organisation. From which of the

following countries does he belong to?

(1) Russia (2) Brazil

(3) France (4) Italy

(5) None of these

Ans: 2

142. Which of the following countries has recently launched super priority same-day visa service for

urgent travelers from India?

(1) Australia (2) New Zealand

(3) Britain (4) US

(5) None of these

Ans: 3

143. National Stock Exchange, the Leading bourse, has launched country’s first dedicated debt

trading platform. Which of the following is/are the functions of the Debt Trading Platform?

A. The debt trading platform is supposed to provide retail investors an opportunity to invest in

corporate bonds on a liquid and transparent exchange platform.

B. Banks and primary dealers are the first to enter and they will provide enough liquidity in the debt

segment.

C. The mutual funds, insurance companies and pension funds are also expected to participate after

guidelines for the same are issued by SEBI.

(1) Only A (2) Only B

(3) Only C (4) All of above

(5) None of these

Ans: 4

144. Recently Union Cabinet approved the National Food Security Bill. Which of the following

statement is/are true regarding this?

A. The amendments to the Bill will guarantee 5 kg of food grains per person per month, while

families in the poorest of the poor will continue to get 35 kg of grains per month.

B. As per the bill around 800 crore people will be entitled to get five kilos of subsidized grain per

month. Rice will be made available 3 Rupees per kilo; wheat will cost 2 rupees a kilo and cereal will

be sold for 1 Rupees per kilo.

C. The food security bill approved is directed towards giving the right to food to around 67 per cent of

India’s 120-crore population.

(1) A and C (2) B only

(3) A only (4) All of above

(5) None of these

Ans: 4

145. Which of the following banks won the prestigious FE India’s Best Banks Award- 2012-13 in

recognition of its strong fundamentals and dynamic growth model?

(1) J & K Bank (2) HDFC Bank

(3) ICICI Bank (4) Axis Bank

(5) None of these

Ans: 1

146. Recently Andhra Pradesh Government launched the AP Industrial Infrastructure Corporation’s

online services for entrepreneurs and grievance redressal system. What is the name of this service?

(1) Parishkaram (2) Sujavyojna

(3) Rashakyojna (4) Gyanyojna

(5) None of these

Ans: 1

147. India Post recently decided to establish third Automated Mail Processing Centre in which of the

following cities?

(1) Rajasthan (2) Hyderabad

(3) Pune (4) Odisha

(5) None of these

Ans: 2

148. Recently how many states and union territories signed tripartite agreements with the Centre

and Bharat Broadband Network to lay optical fibre network for providing broadband services at gram

panchayat level?

(1) 8 states and 2 union territory

(2) 10 states and 3 union territory

(3) 9 states and 1 union territory

(4) 6 states and 2 union territory

(5) None of these

Ans: 3

149. Recently the Union Government accorded approval of 285.75 crore rupees to which of the

following states under PMGSY(Pradhan Mantri Gram Sadak Yojana)?

(1) Himachal Pradesh

(2) Andhra Pradesh

(3) Uttar Pradesh

(4) Bihar

(5) None of these

Ans: 1

150. Recently the Reserve Bank of India announced that it would start the introduction of Plastic

Notes in the market on trial basis. Which of the following statement is/are true regarding plastic

notes?

(1) Plastic bank notes are water proof.

(2) Plastic bank notes are much harder to tear than their paper counterparts.

(3) Plastic bank notes are also called polymer bank notes are made from a plastic polymer.

(4) All of above

(5) None of these

Ans: 4

151. The government recently approved the proposal to hike dearness allowance (DA) of Central

Government Employees to how much per cent from the current 72 per cent?

(1) 82 (2) 85

(3) 80 (4) 75

(5) None of these

Ans: 3

152. Which of the following states bagged awards for best civic management of a tourist

destination?

(1) Andhra Pradesh (2) Kerala

(3) Madhya Pradesh (4) Maharashtra

(5) None of these

Ans: 3

153. Recently India and Liechtenstein signed the Tax Information Exchange Agreement (TIEA).

What is/are the Primary feature/s of TIEA?

A. TIEA is based on international standard of transparency and exchange of information

B. It was decided that the information should be anticipated in context with the administration and

enforcement of the domestic laws of the Contracting Parties in relation with the taxes included in the

agreement.

C. There was also a specific provision that the requested Party will make use of the information

gathering measures to obtain the requested information, despite the fact that the Party does not

need any such information for its tax purposes.

(1) A only (2) B and C

(3) A and C (4) All of above

(5) None of these

Ans: 4

154. Recently three high courts opened in Tripura, Manipur and Meghalaya. With the opening of

these three High Courts in the North Eastern states of India, the total number of High Courts in India

has gone up to………….

(1) 22 (2) 21

(3) 24 (4) 25

(5) None of these

Ans: 3

155. Recently Indian started Visa on Arrival (VoA) facility for the Senior Citizens of Pakistan initiated

at Attari/Wagah Check-Post. This is going to be a single entry Visa and will allow a Pakistani

resident in India for a maximum period of how many days?

(1) 55 (2) 50

(3) 45 (4) 40

(5) None of these

Ans: 3

156. The Union Cabinet recently approved the National Policy for Children, 2012 which recognizes

child survival, health, nutrition, education, development and protection as undeniable rights of every

child. As per the National Child Policy every person below the age of how many years considered as

a child?

(1) 14 (2) 18

(3) 12 (4) 15

(5) None of these

Ans: 2

157. Which of the following countries agreed to co-operate on future missions to the Moon and Mars

after successful collaboration in Chandrayaan-I lunar mission?

(1) India and US

(2) India and China

(3) China and US

(4) US and South Africa

(5) None of these

Ans: 1

158. As per a Swedish think tank, Stockholm International Peace Research Institute’s report China

is the world’s fifth position arm exporter. Which of the following countries is the biggest arm

exporter?

(1) Russia (2) France

(3) US (4) Germany

(5) None of these

Ans: 3

159. World Health Day was recently celebrated across the world. What is the theme of this day

2013?

(1) High Blood Pressure

(2) Diabetics

(3) Low Blood Pressure

(4) Asthma

(5) None of these

Ans: 1

160. Which of the following countries has ranked as the top gold producer for the sixth year in a

row?

(1) Germany (2) South Africa

(3) China (4) India

(5) None of these

Ans: 3

161. ICICI has recently launched its mobile banking service called Mobile Money with which of the

following mobile service providers?

(1) Vodafone (2) Aircell

(3) Tata Docomo (4) Airtel

(5) None of these

Ans: 2

162. Thirty software companies separated From NASSCOM and formed a new association. What is

the name of this new association?

(1) Association of Indian Software and Service Industries (AISSI)

(2) Association of Indian Service and Software Companies (AISSC)

(3) Indian Software Product Industry Round Table (iSpirt)

(4) Indian Association of Software and Services Company (IASSC)

(5) None of these

Ans: 3

163. Finance Minister P. Chidambaram recently launched Rajiv Gandhi Equity Savings Scheme in

Mumbai. Which of the following statement is/are true regarding this scheme?

1. The scheme seeks to provide tax benefits to first-time investors in stock market.

2. Under the scheme, an individual with an income of less than 10 lakh rupees would get tax

incentives for investing up to 50000 rupees in the stock market.

3. As per the notification issued by the SEBI, there would be a lock-in period of one year on

investments made under the scheme.

(1) Only 1 and 2 (2) Only 2 and 3

(3) Only 1 and 3 (4) All of above

(5) None of these

Ans: 4

164. Recently Met Life India Insurance Co. Pvt Ltd merged with which of the following public sector

banks to launch a new joint venture?

(1) State Bank of India

(2) Punjab National Bank

(3) Syndicate Bank

(4) Bank of Baroda

(5) None of these

Ans: 2

165. Which of the following FMCG brand became the First Indian that crossed 5000 crore rupees

mark in terms of retail sales in one year?

(1) Maggi (2) Britania Tiger

(3) Parle-G (4) Dabur

(5) None of these

Ans: 3

166. MCX-SX made its debut as the third National Stock Exchange in India. Which of the following

statement/s is/are true regarding this?

A. MCX-SX begin trading in equities and equity derivatives.

B. MCX-SX index is called SX40.

C. SX40 is the free-float based index, which means that the shares are available at any time for

trading. It is an index of 40 largecap as well as liquid stocks.

D. SX40 includes those companies which have minimum live float of 10 per cent. These companies

are included within top 100 liquid companies.

(1) Both A and D (2) Both A and C

(3) A, B and D (4) All of above

(5) None of these

Ans: 4

167. Which of the following Telecom operator recently closed its operations in Mumbai following a

Supreme Court Order?

(1) Uninor (2) BSNL

(3) Airtel (4) Tata Docomo

(5) None of these

Ans: 1

168. The Cabinet Committee on Economic Affairs (CCEA) has recently gave its principle approval

for the price pooling mechanism of which of the following items?

(1) Gold (2) Sugar

(3) Diesel (4) Coal

(5) None of these

Ans: 4

169. The Union Commerce Ministry of India has just released figures and mentioned that which of

the following industry in India had passed the one billion kilogram mark production?

(1) Tea (2) Spices

(3) Sugar (4) Coffee

(5) None of these

Ans: 1

170. IMF has projected the World GDP growth rate at…………..for 2013, during its announcement

at WEF 2013 at Davos in January, 2013?

(1) 4.5% (2) 3.5%

(3) 4.0% (4) 3.0%

(5) None of these

Ans: 2

171. Congress president Sonia Gandhi has recently launched a kids’ health plan ‘National Child

Health Program’. Which of the following statements is/are true regarding this program?

I. Plan launched is for Rs. 130 crore

II. Plan launched for child health screening and early intervention services for children up to 18 years

of age.

III. The centre launched the plan in Palghar, which will be later extended for the entire

country.

(1) Only I (2) Only II

(3) Only III (4) All of above

(5) None of these

Ans: 4

172. Banks in India traditionally use which system of payment and settlement?

(1) Simultaneous Net Payment (SNP)

(2) Electronic Clearing System (ECS)

(3) National Electronic Fund Transfer (NEFT)

(4) E-Pay

(5) None of these

Ans: 1

173. Who took over the reins of NSE (National Stock Exchange, India’s largest stock exchange on 1

April 2013?

(1) Chitra Ramakrishna

(2) Ramakant Mishra

(3) Manik Mishra

(4) Ketan Mehta

(5) None of these

Ans: 1

174. Market regulator SEBI on 22 April 2013 allowed 47 entities to set up AIFs. What is the meaning

of AIF?

(1) Alternative Investment Funds

(2) Alternative Islamic Funds

(3) Assests and Investment Funds

(4) Analytical Investment Funds

(5) None of these

Ans: 1

175. What is the name of India’s 6th commodity exchange which started its operations on 19 April

2013?

(1) ICEX

(2) Universal Commodity Exchange (UCX)

(3) NCDEX

(4) MCX

(5) NMCE

Ans: 2

176. Concept of Tobin Tax is associated with which field?

(1) Foreign Exchange Transactions

(2) Share Trading

(3) Commodity Trading

(4) Bullion Trading

(5) None of these

Ans: 1

177. Which agency/institution is the regulatory authority for home loans in India?

(1) Housing Development Finance Corporation (HDFC)

(2) National Housing Board (NHC)

(3) National Housing Board (NHC)

(4) Housing Finance Corporation Ltd. (HFCL)

(5) None of these

Ans: 2

178. Which stock exchange recently launched an Islamic equity index especially for Islamic

Investors?

(1) NSE (2) MCX-SX

(3) NCDEX (4) BSE

(5) None of these

Ans: 4

179. In its annual monetary policy statement of 3 May 2013, RBI Governor D. Subbarao highlighted

which measure as the biggest threat to monetary policy?

(1) Widening Current Account Deficit or CAD

(2) Widening Non-Performing Assets

(3) Reducing Forex Reserve

(4) Reducing Gold Price

(5) None of these

Ans: 1

180. What was India’s total indirect tax collection (actual) during 2012-13, as announced on 25 April

2013?

(1) Rs. 2.74 lakh crore

(2) Rs. 3.74 lakh crore

(3) Rs. 4.74 lakh crore

(4) Rs. 5.74 lakh crore

(5) None of these

Ans: 3

181. The Indian Government recently approved the export of extra five million tones

of…………..through private traders.

(1) rice (2) wheat

(3) sugar (4) pulses

(5) None of these

Ans: 2

182. Which telecom company won the 2G spectrum auction in March 2013 offering CDMA services

in eight circles in India?

(1) Tata Tele Services

(2) Reliance Communication

(3) Idea Cellular

(4) Sistema Shyam Telecom Services

(5) Bharti Airtel Telecom

Ans: 4

183. National Thermal Power Corporation’s (NTPC) first overseas coal fired power project is under

construction at:

(1) Maldives (2) Sri Lanka

(3) Myanmar (4) Nepal

(5) None of these

Ans: 2

184. As per the union budget 2013-14, Government of India planned to infuse……..in PSU banks.

(1) Rs. 10000 crore (2) Rs. 11000 crore

(3) Rs. 12000 crore (4) Rs. 13000 crore

(5) Rs. 14000 crore

Ans: 5

185. Mukesh and Anil Ambani have recently signed a Rs. 1200 crore deal for…………….. business.

(1) Textile (2) mining

(3) retail (4) telecom

(5) None of these

Ans: 4

186. In railway budget 2013-14, the railway minister has proposed to run special luxury

coach…………….with the most modern ambience.

(1) Saundarya (2) Sampoorna

(3) Sugam (4) Anubhuti

(5) Sulabh

Ans: 4

187. Government of India has recently cleared the joint venture proposal of Air Asia, a Malaysian

airlines company to launch its services in India in collaboration with:

(1) Jet Airways

(2) Kingfisher Airlines

(3) Tata Sons Airways

(4) Air India

(5) None of these

Ans: 3

188. How many banks were given banking license by the RBI in the second phase in 2003-2004?

(1) 1 (2) 2

(3) 3 (4) 4

(5) 5

Ans: 2

189. Which private bank opened with much fanfare in 90s, collapsed and was later merged with the

Oriental Bank of Commerce in August 2004?

(1) Centurion Bank

(2) Global Trust Bank

(3) South Indian Bank

(4) Times Bank

(5) None of these

Ans: 2

190. When a person opens an account in a bank, the bank is a……………towards the person?

(1) Creditor (2) Debtor

(3) Agent (4) Borrower

(5) None of these

Ans: 2

191. Which act pertains to effectively dealing with the problem of non-performing assets (NPAs) in

Indian banking?

(1) RBI Act

(2) Banking Regulation Act

(3) SARFAESI Act

(4) Banking Ombudsman

(5) None of these

Ans: 3

192. Generally, what is the designation of head of public sector undertakings (PSUs) and PSU

banks in India?

(1) Director General

(2) Chairman

(3) President

(4) Chairman cum Managing Director

(5) None of these

Ans: 4

193. Indian Commercial Banks are categorized into:

(1) Public sector Banks

(2) Foreign Banks

(3) Private Sector Banks

(4) All of the above

(5) None

Ans: 4

194. Which of the following can be done by a nominee after the death of the depositor?

(1) He can renew FDR which has fallen due

(2) He can take premature payment of FDR

(3) He can raise loan against FDR

(4) Both (1) and (2)

(5) None of these

Ans: 2

195. Which of the following persons can not be a Nominee?

(1) Trust

(2) Limited Company

(3) Partnership firm

(4) All of these

(5) None of these

Ans: 4

196. The credit policy of a bank does not consists of?

(1) Lending policies

(2) Quality control

(3) Loan product mix

(4) Advertising of loan products

(5) None of these

Ans: 4

197. Service Area Approach (SAA) is associated With?

(1) Rural and Semi-urban branches of commercial banks

(2) Urban branches of commercial banks

(3) Branches of commercial banks situated in metro cities

(4) Development banks

(5) None of these

Ans: 1

198. Which of the following is not a function of a commercial bank?

(1) Registration of charges and mortgages

(2) Transaction services

(3) Asset transaction

(4) Real-time Gross Settlement

(5) None of these

Ans: 1

199. Which of the following statements is true?

i. Term deposits are non-transaction deposits

ii. Cheques can be issued on short-term deposits

iii. All term deposits are interest bearing deposits

(1) (i) only (2) (ii) only

(3) (iii) only (4) (i) and (iii)

(5) None of these

Ans: 4

200. Which of the following is not a transfer of funds by using the electronic media?

(1) Mail transfer of funds

(2) Telegraph transfer of funds

(3) Electronic credit transfers

(4) Electronic clearing transfers

(5) None of these

Ans: 1





Important & Expected Banking Awareness Qs



1. Which among the following bank has launched the “Smart Vault” first of its kind fully automated digital locker facility in India?



A. SBI B. ICICI

C. HDFC D. AXIS Bank

E. Corporation Bank

Ans (B) India’s largest private sector lender ICICI Bank has launched ‘Smart Vault’ first of its kind fully automated digital locker facility in India. It was launched by Managing Director (MD) and CEO of ICICI Bank, Chanda Kochhar in New Delhi.

2. “Buddy” is a mobile app which aims at providing convenient and secure digital payment system to its customers, launched by which among the following banks?



A. SBI B. ICICI

C. HDFC D. Axis Bank

E. BOB

Ans (A) India’s largest banking entity State Bank of India (SBI) launched a Mobile wallet application dubbed as ‘SBI Buddy’.

3. First Multi- Currency Contactless Card service launched by ____.



A. HDFC Bank B. Axis Bank

C. ICICI Bank D. SBI

E. Yes Bank

Ans (B) Axis Bank has launched the first Contactless Debit, Credit and Multi-Currency Forex Card in association with VISA payWave in the country.

4. Union Government has launched a seven pronged plan called Indradhanush Mission to revamp functioning of ____.



A. Primary Schools in Rural areas

B. To boost Make in India initiative

C. Public Sector Banks (PSBs)

D. Handlooms sector in the country

E. Tourism sector in the country

Ans (C) Union Government has launched a seven pronged plan called Indradhanush Mission to revamp functioning of public sector banks (PSBs). It was launched by Union Finance Minister Arun Jaitley in New Delhi.



5. What is the minimum denomination of Treasury bills to issue in India?



A. Rs. 20,000 B. Rs. 25,000

C. Rs. 30,000 D. Rs. 50,000

E. None of these

Ans (B) Treasury bills are available for a minimum amount of Rs.25,000and in multiples of Rs. 25,000.

At present, the Government of India issues three types of treasury bills through auctions, namely, 91-day, 182-day and 364-day. There are no treasury bills issued by State Governments.

6. In CRAR, A stands for:



A. Application B. Accounts

C. Assets D. Annual

E. Alternate

Ans (C) Assets (Capital to Risk Weighted Assets Ratio)

CRAR is the acronym for capital to risk weighted assets ratio, a standard metric to measure balance sheet strength of banks.

7. _______ is the rate at which banks borrow funds overnight from the Reserve Bank of India (RBI) against approved government securities.



A. CRR B. SLR C. Call Money D. MSF

E. Repo Rate

Ans (D) Marginal Standing Facility (MSF) is a new scheme announced by the Reserve Bank of India (RBI) in its Monetary Policy (2011-12) and refers to the penal rate at which banks can borrow money from the central bank over and above what is available to them through the LAF window.

8. The availability of cash and other cash-like marketable instruments that are useful in purchases and investments are commonly known as:



1) Cash crunch 2) Liquidity

3) Credit 4) Marketability

5) None of these

Ans (2) The availability of cash and other cash-like marketable instruments that are useful in purchases and investments are commonly known as Liquidity.

9. Reserve Bank of India (RBI) has decided to issue _____denomination coins to commemorate International Yoga Day which is being observed every year on 21 June.



A. 10 rupee B. 1 rupee

C. 100 rupee D. 1000 rupee

E. 5 rupee

Ans (A) Reserve Bank of India (RBI) will soon issue 10 rupee denomination coins to commemorate International Yoga Day which is being observed every year on 21 June.

10. According to the recommendation of the 14th Finance Commission, the Share of states in the centre's tax revenue is increased from 32% to _____.



A. 40% B. 42%

C. 45% D. 50%

E. 38%

Ans (B) The Government of India has accepted the recommendation of the 14th Finance Commission which was headed by Dr. Y.V. Reddy. The share of states in the centre's tax revenue is increased from 32% to 42%.

11. RBI grants “in-principle” approval to Reliance Industries Limited to set up Payments Banks in the country. To which among the following RIL has partnered with for Payment Bank Licence?



A. LIC B. SBI

C. Bajaj Capital D. Apollo Munich

E. None of these

Ans (B) Reliance partnered with SBI (State Bank of India) for Payment Bank Licence. RIL will be the promoter and SBI will be the joint venture partner with equity investment of up to 30%.

12. National Electronic Funds Transfer (NEFT) is a nation-wide payment system facilitating one-to-one funds transfer. What is the limit on the amount that could be transferred using NEFT?



A. Rs. 2 lakh B. Rs. 5 lakh

C. Rs. 10 lakh D. Rs. 1 lakh

E. No limit

Ans (E) There is no limit – either minimum or maximum – on the amount of funds that could be transferred using NEFT. However, maximum amount per transaction is limited to Rs.50,000/- for cash-based remittances within India and also for remittances to Nepal under the Indo-Nepal Remittance Facility Scheme.

13. What is the maximum value to be stored in a prepaid card?



A. No limit B. Rs. 10,000

C. Rs. 5,000 D. Rs. 50,000

E. Limits depends on the issued by banks and authorised non-bank entities

Ans (D) As per extant instructions, the maximum value that can be stored in any prepaid card (issued by banks and authorised non-bank entities) at any point of time is Rs 50,000/-



14. Reserve Bank of India (RBI) has constituted a committee to prepare a 5 year action plan to spread the reach of financial services across country to unbanked population. The committee will be headed by ____.



A. Nachiket Mor B. KC Chakraborty

C. Anand Siha D. Urjit Patel

E. Deepak Mohanty

Ans (E) The committee will be headed by RBI executive director Deepak Mohanty. It will review the existing policy of financial inclusion, including customer protection framework and supportive payment system.

15. Money lent for one day is called:



(1) Call money (2) Notice money

(3) Term Money (4) All of the above

(5) None of these

Ans (A) ‘Call Money’ is the borrowing or lending of funds for 1day. Where money is borrowed or lend for period between 2 days and 14 days it is known as ‘Notice Money’. And ‘Term Money’ refers to borrowing/lending of funds for period exceeding 14 days.

16. Treasury bills or T-bills, which are money market instruments, are short term debt instruments issued by ____.



A. Reserve Bank of India B. Individual Banks

C. Government of India D. IRDA

E. Corporate & Financial Institutions

Ans (C) Treasury bills or T-bills, which are money market instruments, issued by the Government of India and are presently issued in three tenors, namely, 91 day, 182 day and 364 day. Treasury bills are zero coupon securities and pay no interest.

17. India's first International Financial Services Centre opened at _____.



(1) Gujarat (2) West Bengal

(3) Maharashtra (4) Rajasthan

(5) None of these

Ans (A) India's first International Financial Services Centre set up in Gujarat International Finance Tec-City (GIFT).

18. A Reserve Bank of India (RBI) committee has recommended conversion of Urban Cooperative Banks (UCBs) with business size of 20,000 crore rupees or more into regular banks. This recommendation was given by the High Powered Committee on UCBs headed by ___.



A. Deepak Mohanty B. Urjit Patel

C. MB Shah D. R Gandhi

E. None of these

Ans (D) The High Powered Committee on UCBs headed by RBI Deputy Governor R Gandhi.

19. The Public Provident Fund is savings-cum-tax-saving instrument in India, introduced by the National Savings Institute of the Ministry of Finance in 1968. What is the minimum amount to be invested in PPF account?



A. Rs. 100 B. Rs. 500

C. Rs. 1000 D. Rs. 200

E. None of these

Ans (B) The minimum investment of Rs. 500 and the maximum amount is Rs. 1.5 lakh per annum. RATE OF INTEREST - 8.70%

20. The transitional period for full implementation of Basel III Capital Regulations in India is extended upto?



(1) March 1, 2018 (2) March 31, 2019

(3) April 1, 2018 (4) April 30, 2018

(5) None of these

Ans (B) The transitional period for full implementation of Basel III Capital Regulations in India is extended upto March 31, 2019, instead of as on March 31, 2018.

21. RBI has increased the limit for foreign exchange (FOREX) remittances under Liberalised Remittance Scheme (LRS) from $125,000 to _____ per person per year.



A. $250,000 B. $150,000

C. $130,000 D. $20,000

E. $145,000

Ans (A) RBI has increased the limit for foreign exchange (FOREX) remittances under Liberalised Remittance Scheme (LRS) from $125,000 to $250,000 (Rs. 1.5 crore) per person per year.

22. According to the guidelines released by RBI, a Payments banks can open small savings accounts and accept deposits of up to ___.



A. 10 lakh B. 5 lakh

C. 3 lakh D. 2 lakh

E. None of these

Ans (D) According to the guidelines released by RBI, Payments banks can open small savings accounts and accept deposits of up to Rs.1 lakh per individual customer.

23. Which bank has launched “Saral-Rural Housing Loan’ scheme for weaker sections of society?



A. Axis Bank B. SBI

C. HDFC D. Yes

E. ICICI

Ans (E) ICICI Bank has launched ‘ICICI Bank Saral-Rural Housing Loan’ Scheme for weaker sections of society. Housing loan will be provided at the ICICI Bank





Base Rate (known as I-Base) which is currently at 9.70%.

24. What is the minimum capital requirement for New Banks in the Private Sector



A. Rs. 200 crore B. Rs. 300 crore

C. Rs. 500 crore D.Rs. 600 crore

E. None of these

Ans (C) Minimum capital requirement for New Banks in the Private Sector is Rs. 500 crore.

25. Many times we read a term ‘ECB’ in financial newspapers. What is the full form of ECB?



A. Essential Commercial Borrowing

B. Essential Credit and Borrowing

C. External Credit and Business

D. External Commercial Borrowing

E. None of the above

Ans. (D)An external commercial borrowing (ECB) is an instrument used in India to facilitate the access to foreign money by Indian corporations and PSUs (public sector undertakings).

26. Which among the following banks had launched 'Dhanchayat', an educational film to raise awareness on the dangers of borrowing money from unorganised sources?



1) Axis 2) ICIC

3) HDFC 4) SBI

5) YES

Ans (3) HDFC Bank Ltd. launched 'Dhanchayat' an educational film to raise awareness on the dangers of borrowing money from unorganised sources. This film has been launched under the aegis of Swachch Banking, the Bank's CSR initiative for rural India.

27. Which among the following has launched a Smart Humsafar Plan, a unique plan which offers multiple benefits of savings and insurance cover for husband and wife under a single policy? 1) ICICI Prudential 2) SBI Life 3) NIACL 4) LIC 5) HDFC ERGO



Ans (2) Private insurer SBI Life Insurance launched Smart Humsafar, a unique plan which offers multiple benefits of savings and insurance cover for husband and wife under a single policy.

28. What is the maximum monthly pension for the subscribers under Atal Pension Yojana?



(1) Rs 2000 (2) Rs 3000

(3) Rs 4000 (4) Rs 5000

(5) None of these

Ans (4) The Atal Pension Yojana (APY) is open to all Indians between the age of 18 and 40. Under the scheme, the minimum monthly pension for the subscribers ranging between Rs 1000 and Rs 5000 per month.

29. What is the minimum amount to be invested under Sukanya Samriddhi Scheme?



A. Rs. 500 B. Rs. 1000

C. Rs. 1500 D. No ceiling

E. Rs. 100

Ans (B) Rs. 1000 is the minimum and Rs. 1.5 lakh is the maximum amount to be invested under Sukanya Samriddhi Scheme.

30. Under Pradhan Mantri Jan-Dhan Yojana (PMJDY), the accidental insurance cover of _____will be provided to all the beneficiaries.



A. Rs.1.00 lac B. Rs.2.00 lac

C. Rs.3.00 lac D. Rs.30,000

E. Rs.50,000

Ans (A) Under Pradhan Mantri Jan-Dhan Yojana (PMJDY), the accidental insurance cover of Rs. 1 lakh will be provided to all the beneficiaries. Life insurance cover of Rs.30,000.

31. The Reserve Bank of India advised banks to make the Know Your Customer (KYC) procedures mandatory while opening and operating the accounts. RBI has issued the KYC guidelines under Section 35 (A) of the ____.



A. Banking Regulation Act, 1949

B. RBI Act, 1934

C. Negotiable Instruments Act, 1881

D. Banking Regulation Act, 1935

E. Both A & B

Ans (A) RBI has issued the KYC guidelines under Section 35 (A) of the Banking Regulation Act, 1949 and contravention of the same will attract the penalties under the relevant provisions of the act.

32. At which rate the RBI lends money to a public sector bank on a long term basis?



A. PLR B. CRR

C. Repo Rate D. Reverse Repo Rate

E. Bank Rate

Ans (E) Bank rate is the rate of interest implemented by RBI when it lends money to a public sector bank on a long term basis, i.e. from a period ranging from 90 days to 1 year

Note: Repo Rate is the rate at which RBI lends money to commercial banks in the event of any shortfall of funds. It is the rate of interest which RBI implements on the short term loans, i.e., from a period ranging between 2 days to 3 months (90 Days).







33. India’s gross domestic product (GDP) grew at what percent in 2014-15. The growth is based on new methodology with 2011-12 as base year.



A. 5.1 % B. 7.3 %

C. 7.6 % D. 6. %

E. 5.5 %

Ans (B) India's gross domestic product (GDP) grew 7.3 per cent in 2014-15, slightly less than the advance estimate of 7.4 per cent. The growth is based on a new methodology of calculating GDP, with 2011-12 as the base year, against 2004-05 used earlier.

34. What is the full form of PPP?



A. Purchasing Power Parity

B. Public Private Parity

C. Primary Power Parity

D. Personal Private Power

E. None of these

Ans (A) Purchasing power parity (PPP) is a component of some economic theories and is a technique used to determine the relative value of different currencies.

35. Which of the following is the first North East state to achieve 100 percent success in implementing the Pradhan Mantri Jan Dhan Yojana?



A. Nagaland B. Tripura

C. Meghalaya D. Assam

E. None of these

Ans: (C) Meghalaya became the first state in India to achieve 100 percent success in implementing the Pradhan Mantri Jan Dhan Yojana.

36. What is the name given to Union Finance Ministry’s flagship project which aims at widening the tax base by catching tax evaders using technology?



(1) Project Insight (2) Indradhanush

(3) Gospel Missions (4) Jan jagran

(5) None of these

Ans (1) Project Insight is an ambitious project of the Union Finance Ministry to bring into net black money hoarders and tax evaders by using optimum technology. It will primarily track the Permanent Account Numbers (PAN) being quoted on financial transactions and tally them with income tax filings

37. What is the Minimum amount of transfer required for RTGS is _____.



A. Rs. 5 lakh B. Rs. 2 lakh

C. Rs. 1 lakh D. No limit

E. None of these

Ans (B) Minimum amount of transfer required for RTGS (Real Time Gross Settlement) is Rs. 2 lakh. There is no upper limit, though. The beneficiary bank has to credit the beneficiary's account within two hours of receiving the funds transfer message.

38. Who is the sole authority to issue and manage currency in India?



A. Government of India B. Reserve Bank of India

C. Finance Ministry D. State Bank of India

E. None of these

Ans: (B) In terms of Section 22 of RBI is the sole authority for issue of currency in India.

39. Which among the following bank has opened its branch in Shanghai, China?



A. SBI B. ICICI

C. HDFC D. AXIS Bank

E. Corporation Bank

Ans (B) To help Indian and Chinese corporates in cross-border businesses, top private sector lender ICICI Bank opened its first branch in Shanghai, China, which was inaugurated by Prime Minister Narendra Modi.

40. In CRR, C stands for __.



A. Capital B. Currency

C. Core D. Cash

E. None of these

Ans (D) Cash reserve Ratio (CRR) is the amount of funds that the banks have to keep with the RBI. Current CRR is 4%.

41. MICR code is used for ___.?



(1) For Electronic Clearance of Cheques

(2) For Electronic Funds Transfer

(3) For Code Banking Solution

(4) For Cheque Truncation Services

(5) None of these

Ans (A) Magnetic Ink Character Recognition Code (MICR Code) is a character-recognition technology used mainly by the banking industry to ease the processing and clearance of cheques and other documents.

42. The Statutory Liquidity Ratio (SLR), the amount of liquid assets such as cash, precious metals and other short-term securities are kept with:



A. RBI B. Individual banks

C. Finance Ministry D. A bank designated by RBI

E. None of these

Ans (B) Statutory liquidity ratio (SLR) is the Indian government term for reserve requirement that the commercial banks in India require to maintain in the form of cash, gold, government approved securities before providing credit to the customers.



43. Which country launched world first Facial recognition ATM?



A. China B. Japan

C. US D. Canada

E. Brazil

Ans (A) China has unveiled the world’s first facial recognition ATM. The new cash machine was developed by Tsinghua University, a research university in Beijing.

44. A cheque is considered a valid cheque if the date entered on the cheque is within ______of the actual date on which it is presented.



A. 6 months B. 3 months

C. 9 months D. 1 months

E. None of these

Ans (B) A cheque is considered a valid cheque if the date entered on the cheque is within 3 months of the actual date on which it is presented. For example, a cheque with the date of 10th Jan 2015 will be valid if it is presented to the bank on or before 10th April 2015.

45. As per the Banking Ombudsman Scheme, any person can file a complaint before the Banking Ombudsman, if the satisfied reply is not received from the bank within a period of ____.



A. one month B. three months

C. two months D. six months

E. None

Ans (A) One can file a complaint before the Banking Ombudsman if the reply is not received from the bank within a period of one month after the bank concerned has received one s representation, or the bank rejects the complaint, or if the complainant is not satisfied with the reply given by the bank.

46. The headquarter of BRICS Bank “The New Development Bank (NDB)” located in ___.



A. Beijing, China B. Shanghai, China

C. New Delhi, India D. New York, US

E. None of these Ans (B) The New Development Bank (NDB) headquarter located in Shanghai, China.

47. Money lent for more than one day but less than 15 days in the money market is known as:



A. Notice Money B. Call Money

C. Term Money D. All of the above

E. None of the above

Ans (A) The period of lending may be for a period of 1 day which is known as call money and between 2 days and 14 days which is known as notice money. Term money refers to borrowing/lending of funds for a period exceeding 14 days.

48. National Electronic Funds Transfer (NEFT) is a nation-wide payment system facilitating one-to-one funds transfer. What is the limit on the amount that could be transferred using NEFT?



A. Rs. 2 lakh B. Rs. 5 lakh

C. Rs. 10 lakh D. Rs. 1 lakh

E. No limit

Ans (E) There is no limit – either minimum or maximum – on the amount of funds that could be transferred using NEFT. However, maximum amount per transaction is limited to Rs.50,000/- for cash-based remittances within India and also for remittances to Nepal under the Indo-Nepal Remittance Facility Scheme.

49. Smart Money’ is a term used for –



A. Internet Banking B. FDRs in Banks

C. Credit Card D. Demand Drafts of Banks

E. None of these

Ans (C) SMART Money term used for Credit Card, is an electronic wallet, that allows you to Shop, pay bills, etc.

50. The Indian Financial System Code (IFS Code) is an alphanumeric code that uniquely identifies a bank-branch participating in the two main Electronic Funds Settlement Systems in India. IFSC code consists of how many alphanumeric codes?



A. 12 B. 11

C. 10 D. 9

E. 8

Ans (B) IFSC is an 11- alphanumeric code with the first four alphabetic characters representing the bank name, and the last six characters (usually numeric, but can be alphabetic) representing the branch. The fifth character is 0 (zero) and reserved for future use. Bank IFS Code is used by the NEFT & RTGS systems to route the messages to the destination banks/branches.

51. Which among the following committee constituted on Payment Banks license?



(A) Dr. Nachiket Mor (B) R. Gandhi

(C) Anand Sinha (D) Deepak Mohanty

(E) None of these

Ans (A)Payment Banks applicant applications are analyzed and evaluated by an External Advisory Committee (EAC). The EAC Committee for Payment Banks is chaired by Dr. Nachiket Mor, Director, Central Board of the Reserve Bank of India.

52. The Reserve Bank has revised priority sector lending norms to ensure a phased increase in loans to small farmers. As per the new guidelines, credit to small and marginal farmers should be at least how much per cent of a bank's total credit by March 2017?



A. 5 percent B. 6 percent







C. 7 percent D. 8 percent

E. 9 percent

Ans (D) The Reserve Bank has revised priority sector lending norms to ensure a phased increase in loans to small farmers, and also made it clear that smaller foreign banks will have to meet the 40 percent target over the next five years. As per the new guidelines, credit to small and marginal farmers should be at least 8 percent of a bank's total credit by March 2017.

53. The initial total committed resources of the Contingent Reserve Arrangement (CRA) shall be $100 billion in BRICS Development Bank. What is the total contribution of South Africa?



A. $18 billion B. $10 billion

C. $5 billion D. $41billion

E. None of these

Ans (C) In BRICS Development Bank, the initial total committed resources of the Contingent Reserve Arrangement (CRA) is $100 billion, with the following individual commitments: China – $ 41 billion; Brazil – $18 billion; Russia – $18 billion; India – $18 billion and South Africa – $5 billion.

54. With which of the following is SARFAESI Act 2002 related?



(1) Recovery of bad loans

(2) Regulation of foreign exchange

(3) Fixation of interest rates

(4) Acquisition of small banks

(5) None of these

Ans (1) The Securitisation and Reconstruction of Financial Assets and Enforcement of Security Interest Act, 2002 (SARFAESI) empowers Banks / Financial Institutions to recover their non-performing assets (bad loans) without the intervention of the Court.

55. Which among the following bank has tied up with Chinese e-commerce giant Alibaba to provide easier trade finance to small and medium enterprises (SMEs) in the country?



(1) HDFC Bank (2) SBI Bank

(3) Canara Bank (4) ICICI Bank

(5) Axis Bank

Ans (4)In order to tap into the booming e-commerce market in India, ICICI Bank, the country’s largest private sector lender, has tied up with Chinese e-commerce giant Alibaba to provide easier trade finance to small and medium enterprises (SMEs) in the country.

56. Who among the following is the head of the committee constituted on the controversial issue of payment of Minimum Alternate Tax (MAT) by foreign institutional investors?



(1) Vijay Kelkar (2) SS Ahluwallia

(3) RK Narayna (4) HD Devraj

(5) AP Shah

Ans (5) The Justice AP Shah committee looking into the issue of levying minimum alternate tax (MAT) on foreign portfolio investors (FPI).

57. The govt scheme, which gives accidental deathcumdisability cover of Rs two lakh to all the savings bank account holders in the age group of 18 to 70 years for a premium of Rs 12 per annum, is



1) Pradhan Mantri Jivan Jyoti Bima Yojana (PMJJBY)

2) Pradhan Mantri Suraksha Bima Yojana (PMSBY)

3) Janashree Bima Yojana (JBY)

4) Aam Admi Bima Yojana (AABY)

5) None of these

Ans(2) Pradhan Mantri Suraksha Bima Yojana (PMSBY) - Account holders between 18 years (completed) and 70 years (age nearer birthday) who give their consent to join / enable auto-debit, as per the modality, will be enrolled into the scheme. Premium: Rs. 12 per annum. Risk Coverage: Total coverage (sum-insured) under the scheme is Rs. 2 Lakh.

58. What is the full form of the term NDTL, as used in banking environment?



(1) Net Demand and Term Liability

(2) Net Demand and Time Liability

(3) Net Demand and Term Liquidity

(4) Net Demand and Time Liquidity

(5) None of the above

Ans (2) NDTL full form is - Net Demand and Time Liability

It is a sum of demand and time liabilities (deposits) of banks with public and other banks wherein assets with other banks is subtracted to get net liability of other banks. Deposits of banks are its liability and consist of demand and time deposits of public and other banks.

59. Magnetic Ink Character Recognition (MICR) is a 9 digits code is used mainly by the banking industry to ease the processing and clearance of cheques and other documents. The first three digits represent the _____.



(1) City (2) Bank

(3) Branch (4) State

(5) None of these

Ans (1) | Magnetic Ink Character Recognition Code (MICR Code) is a character-recognition technology used mainly by the banking industry to ease the processing and clearance of cheques and other documents.





Note: MICR code consists of 9 digits. First 3 digits represent the city, next 3 digits represent the bank and the last 3 digits represent the branch.

60. The implementation of Goods and Services Tax (GST) is expected by



A. April 2016 B. January 2016

C. April 2017 D. March 2016

E. January 2017

Answer (A) |Implementation of Goods and Services Tax is expected by April 2016.

61. Global Rating Agency Fitch has downgraded Viability Rating (VR) by one notch to 'bb' of which among the following banks?



A. Bank of Baroda B. State Bank of India

C. Punjab National Bank D. Canara Bank

E. IDBI Bank

Answer (C) Fitch on 2 September 2015, downgraded Punjab National Bank’s (PNB’s) viability rating — a parameter to measure creditworthiness — by a notch to ‘bb’.

62. Union Cabinet approved how much per cent FDI in white label ATMs through automatic route?



A. 100 percent B. 74 percent

C. 51 percent D. 49 percent

E. 25 percent

Answer (A) The Union Cabinet on 9 September 2015 approved 100 percent Foreign Direct Investment (FDI) in white label Automated Teller Machines (WLAs).





63. Central Statistical Office (CSO) released the estimates of Gross Domestic Product (GDP) for the April-June quarter of 2015. As per the estimates, India grew by how much per cent during the first quarter of the 2015-16 financial year?

A. 7.5 percent B. 7 percent

C. 8 percent D. 7.6 percent

E. 8. Percent

Answer (B) Central Statistical Office (CSO) on 31 August 2015 released the estimates of Gross Domestic Product (GDP) for the April-June quarter of 2015. As per the estimates, India grew by 7 percent during the first quarter of the 2015-16 financial year.

64. Which of the following countries decided to end the local currency by September 2015?



A. Ethiopia B. Kenya

C. Sudan D. Zimbabwe

E. Senegal

Answer (D) Zimbabwe has decided to end the local currency (Zimbabwean Dollar) by September 2015. This is due to the fact that the Zimbabwean dollar has become almost worthless.

65. What is the name of the platform launched by State Bank of India (SBI) that enables customers to book their foreign exchange transactions online?



A. SBI eforex B. SBI emoney

C. SBI eonline D. SBI eshare

E. SBI ebuddy

Answer (A) State Bank of India launched “SBI eforex”, an internet-based platform that enables customers to book their foreign exchange transactions online.

66. India has become the second largest player in the China-led Asian infrastructure and Investment Bank (AIIB). What is India’s stake in the bank’s shares in AIIB?



A. 7.5% B. 8.52%

C. 20.06% D. 30.34%

E. 5.2%

Answer (B) Bank share - China has received a 30.34 percent stake in the bank’s shares; India and Russia have 8.52 and 7.5 percent respectively. Voting share - India has become the second largest player in the China-led Asian infrastructure and Investment Bank (AIIB), with a 7.5 percent voting share, while China and Russia gained 20.06 percent and 5.2% respectively.

67. A savings as well as current account should be treated as inoperative / dormant if there are no transactions in the account for over a period of _____ years.



A. One B. Two

C. Three D. Four

E. Five

Answer (B) Dormant means inactive and inoperative means which is not being operated i.e. no transactions have been undertaken recently. In terms of RBI guidelines "A savings as well as current account should be treated as inoperative / dormant if there are no transactions in the account for over a period of two years".

68. Reserve Bank of India (RBI) signed a Memorandum of Understanding (MoU) with which country central bank on Supervisory Cooperation and Exchange of Supervisory Information?



A. Nepal Rastra Bank (NRB)

B. Central Bank of Sri Lanka

C. Reserve Bank of Australia

D. Bangladesh Bank

E. Royal Monetary Authority of Bhutan





Answer (A) Reserve Bank of India (RBI) on signed a Memorandum of Understanding (MoU) with the Nepal Rastra Bank (NRB) on Supervisory Cooperation and Exchange of Supervisory Information.

69. According to the World Bank Report, which among the following states has topped in ease of doing business in India?



A. Maharashtra B. Gujarat

C. Andhra Pradesh D. Madhya Pradesh

E. Tamil Nadu

Answer (B) Gujarat has been placed to top position in World Bank’s first ever ranking of States on the ease of doing business in India. All States were assessed over a six-month period from January to June 2015 on the basis of percentage scores in a 98-point reforms agenda. Top 5 States: Gujarat (71.14%), Andhra Pradesh (70.12%), Jharkhand (63.09%), Chhattisgarh (62.45%) and Madhya Pradesh (62.00%).

70. In CDR, “R” stands for ____.



A. Ratio B. Receipt

C. Regulation D. Restructuring

E. Reserve

Answer (D) Corporate Debt Restructuring (“CDR”) mechanism is a voluntary non statutory mechanism under which financial institutions and banks come together to restructure the debt of companies facing financial difficulties due to internal or external factors, in order to provide timely support to such companies.

71. A measurement of a country’s trade in which the value of goods and services it imports exceeds the value of goods and services it exports is known as _____.



A. Current Account Deficit B. Revenue Deficit

C. Fiscal Deficit D. Capital Account

E. Balance of Payments

Answer (A) Current Account Deficit - A measurement of a country’s trade in which the value of goods and services it imports exceeds the value of goods and services it exports.

72. The Bank for International Settlements (BIS) is the world's oldest international financial organization established on 17 May 1930. The mission of the BIS is to serve central banks in their pursuit of monetary and financial stability, to foster international cooperation in those areas and to act as a bank for central banks. The head office of BIS is located in ___.



A. Hong Kong B. Switzerland

C. China D. Mexico

E. Australia

Answer (B) The Bank for International Settlements (BIS) is the world's oldest international financial organization established on 17 May 1930. The head office is in Basel, Switzerland.

73. What is the minimum amount required to open a Fixed Deposit (FD)?



A. Rs. 100 B. Rs. 500

C. Rs. 1000 D. Rs. 5000

E. No limit

Answer (C) A fixed deposit (FD) is also known as Term Deposit a financial instrument provided by banks which provides investors with a higher rate of interest than a regular savings account, until the given maturity date. The minimum deposit amount required for the FD account is Rs 1,000. There is no fixed multiple or upper limit. FD can be opened for a minimum period of 7 days up to maximum period of 10 years.

74. What is the maximum deposit amount insured by the Deposit Insurance and Credit Guarantee Corporation (DICGC)?



A. Rs. 10 lakh B.Rs. 5 lakh

C.Rs. 1 lakh D.Rs. 50,000

E. None

Answer (C) Deposit Insurance and Credit Guarantee Corporation ( DICGC) is a subsidiary of Reserve Bank of India. DICGC insures all bank deposits, such as saving, fixed, current, recurring deposits for up to the limit of Rs. 100,000 of each deposits in a bank

75. If one is not satisfied with the decision passed by the Banking Ombudsman, one can approach the appellate authority to ____.



A. Concern Bank Branch Manager

B. Deputy Governor of the RBI

C. General Manager of Concern Bank

D. Finance Secretary

E. General Manager of RBI

Answer (B) If one is not satisfied with the decision passed by the Banking Ombudsman, one can approach the appellate authority who is the Deputy Governor of the RBI.

76. The Reserve Bank of India recently paid how much amount of dividend to the government, which is considered to be the highest ever dividend paid from the central bank in its 80-year history, and 22% more than it paid last year?



A. Rs. 33,010 crore B. Rs 66,000 crore

C. Rs. 25,020 crore D. Rs. 55,000 crore

E. None of these

Answer (B) The Reserve Bank of India paid a dividend of nearly Rs 66,000 crore to the government, the highest ever from the central bank in its 80-year history, and 22% more than it paid last year. On a point





to point basis, RBI's dividend payment to the government is up more than four times in as many years.

77. The revised Financial Sector Legislative Reforms Commission (FSLRC) report which was released by the Finance Ministry proposes to subsume a number of existing sector specific financial regulators (SEBI, IRDA, PFRDA, FMC) into a Unified Financial Authority (UFA). FSLRC is headed by ____.



A. BN Srikrishna B. B. Sudershan Reddy

C. A.K. Mathur D. A.K. Ganguly

E. B.S. Chauhan

Answer (A) Financial Sector Legislative Reforms Commission (FSLRC) was set up by the Indian Government in pursuance of the announcement made in Union Budget 2010-11, to help rewriting and harmonizing the financial sector legislation, rules and regulations so as to address the contemporaneous requirements of the sector. The Commission is chaired by Supreme Court Justice (Retired) B. N. Srikrishna, and had ten members with expertise in the fields of finance, economics, law and other relevant fields

78. National Investment and Infrastructure Fund (NIIF) is a fund created by the Government of India for enhancing infrastructure financing in the country. To be operational by the year end, the headquarter of NIIF will be located in ____.



A. Mumbai B. New Delhi

C. Bengaluru D. Chennai

E. Kolkata

Answer (A) NIIF, proposed to be set up as a Trust, would raise debt to invest in the equity of infrastructure finance companies such as Indian Rail Finance Corporation (IRFC) and National Housing Bank (NHB). The idea is that these infrastructure finance companies can then leverage this extra equity, manifold. In that sense, NIIF is a banker of the banker of the banker.

79. The committee on Urban Cooperative Bank (UCB) which is headed by R. Gandhi (Deputy Governor of RBI) has recommended that UCBs with business size more than ______ be allowed to convert into universal commercial banks.



A. Rs. 20,000 cr B. Rs. 25, 000 cr

C. Rs. 50,000 cr D. Rs. 10,000 cr

E. Rs. 5,000 cr

Answer (A) To overhaul and strengthen the urban cooperative bank (UCB) structure, an internal committee of the Reserve Bank of India (RBI) chaired by deputy governor R Gandhi has recommended that UCBs with business size more than Rs 20,000 crore be allowed to convert into universal commercial banks while those with a lower size be converted to small finance banks.

80. The Global Investor Meet 2015 held in ____.



A. New Delhi B. Mumbai

C. Bengaluru D. Chennai

E. Ahmedabad

Answer (D) The Global Investors Meet (GIM 2015) was on 10 September 2015 held in Tamil Nadu to woo investors to invest in the state. The GIM 2015 was officially inaugurated by Tamil Nadu Chief Minister J Jayalalithaa at the Chennai Trade Centre.

81. Special drawing rights (SDRs) are international foreign exchange reserve assets issued by-



A. World Bank B.WTO

C. ADB D. IMF

E. None of these

Answer (D) The Special Drawing Right (SDR) is an international reserve asset, created by the International Monetary Fund (IMF) in 1969 to supplement the existing official reserves of member countries.

82. In Deposits Account “KYC” (Know Your Customer) has been implemented in 2002 as per directive of –



A. IBA B. RBI

C. Ministry of Finance D.SEBI

E. IRDAA

Answer (B) KYC means “Know Your Customer”. It is a process by which banks obtain information about the identity and address of the customers. KYC has been implemented in 2002 as per directive of Reserve Bank of India.

83. Which of the following indexes used as the key measure of inflation in India?



A. Wholesale Price Index

B. Consumer Price Index

C. Sensex and Nifty

D. Interest rate offered by banks on deposits

E. None of these

Answer (B) The Central Bank of India (RBI) had adopted the new Consumer Price Index (CPI) as the key measure of inflation. Earlier, RBI had given more weightage to Wholesale Price Index (WPI) than CPI as the key measure of inflation for all policy purposes.



84. A major Public Sector Bank has decreased the interest rates on loans by 25 basis points – was news in some major financial newspaper recently. This means bank has decreased interest rate by 25 basis points of ___.



A. Saving Bank Interest Rate

B. Base Lending Rate

C. Repo Rate

D. Present Rate on Deposits

E. Discounted Rates of Interest

Answer (B) RBI has reduced the policy rate by 75 basis points since the beginning of this calendar year while most banks have reduced base lending rates by just 25 basis points.

85. What is the rank of India in the World Bank's 'Ease of Doing Business 2015' report?



A. 134 B. 141

C. 142 D. 135

E. 145

Answer (C) India is ranked 142 among 189 nations in the World Bank's 'Ease of Doing Business 2015' report.

86. Which among the following was the venue for the Asian Banker Summit 2015?



A. Hong Kong B. Shanghai

C. New Delhi D. Dhaka

E. None of these

Answer (A) The 16th Asian Banker Summit was held in Hong Kong. The Asian Banker Summit, is the region’s largest annual meeting of global thought leaders and senior industry practitioners, it is the foremost forum for engaging on global themes bringing together global thought leaders and industry specialists.

87. Who among the following names as the first President of New Development Bank of BRICS nations?



A. Arun Poorie B. Deepak Parekh

C. KV Kamath D. Indra Nooyi

E. Ravi Khanna

Answer (C) K. V. Kamath appointed as the first head of the new development bank the BRICS group.

88. According to the Moody's Investors Service, India ranks among the top five sovereign debt issuers from the emerging economies as of end-2014. What is the rank of India in the list?



A. First B. Second C. Third

D. Fourth E. Fifth

Answer (B) India ranks among the top five sovereign debt issuers from the emerging economies after China, said Moody's Investors Service. "The top five largest sovereign EM (emerging market) debt issuers, as of end-2014, were

1. China (with $3.5 trillion total sovereign debt outstanding),

2. India ($1.3 trillion),

3. Brazil ($1.2 trillion),

4. Mexico ($387.5 billion) and

5. Turkey ($265.5 billion),".

89. Recently, global financial services firm Morgan Stanley has revised the India FY16 growth outlook to ___ from 7.9%



A. 7.6% B. 7.5% C. 7.3%

D. 7.1% E. 6.5%

Answer (B) Morgan Stanley revises FY16 growth outlook to 7.5% from 7.9%.

90. As far as commitments to invest in India, United Arab Emirates (UAE) has promised to invest how much billion dollar in Infrastructure Investment Fund?



A. $35-billion B. $41 billion

C. $20 billion D. $50 billion

E. $75-billion

Answer (E) As far as commitments to invest in India, in the past year, the US has committed $41 billion, Japan has laid out a $35-billion plan, China $20 billion, and now the biggest of all, $75 billion, has been promised by the United Arab Emirates (UAE) as its investment target for India.

91. The government and the Reserve Bank have reached a consensus on the structure for the proposed MPC and Public Debt Management Agency (PDMA), on deciding on interest rate on advice of technical advisory committee. What is the expand form of MPC?



A. Monetary Policy Control

B. Management Public Committee

C. Monetary Policy Committee

D. Market Policy Company

E. None of these

Answer (C) The government and the Reserve Bank have reached a consensus on the structure for the proposed Monetary Policy Committee and Public Debt Management Agency.

92. The SARFAESI Act 2002 empowers Banks / Financial Institutions to recover their non-performing assets (bad loans) without the intervention of the Court. In “SARFAESI” second “S” stands for ____.



A. Securitisation B. Security

C. Statutory D. Share

E. Saving

Answer (B) In “SARFAESI” second “S” stands for – Security. The Securitisation and Reconstruction of Financial Assets and Enforcement of Security Interest Act, 2002 (SARFAESI) empowers Banks / Financial





Institutions to recover their non-performing assets (bad loans) without the intervention of the Court.

93. The Reserve Bank of India (RBI) doubled the limit for cash withdrawal at points of sale (PoS) in tier III to VI cities to____.



A. Rs. 1000 B. Rs. 2000 C. Rs. 4000

D. Rs. 6000 E. Rs. 10,000

Answer (B) The Reserve Bank of India (RBI) doubled the limit for cash withdrawal at points of sale (PoS) in tier III to VI cities from Rs1,000 to Rs2,000 per day. The facility will be available for debit cards and open system pre-paid cards issued by banks. The limit remains unchanged at Rs1,000 per day for tier I and II cities.

94. The mobile banking app “LIME” has been launched by which of the following banks?



A. SBI B. HDFC C. ICICI

D. PNB E. AXIS

Answer (E) The mobile banking app “LIME” has been launched by Axis Bank. The app will allow customers in both account and non-account holders to compare and shop, split bills, transfer money and even share their mobile banking wallets. This app can also be used to pool money to buy gifts or for group travel by friends.

95. Many a times we read in the newspapers that RBI has changed or revised a particular ratio/rate by a few basis points. What is basis point?



A. Ten per cent of one hundredth point

B. One hundredth of 1%

C. One hundredth of 10%

D. Ten per cent of 1000

E. None of these

Ans (B) Basis points - One hundredth of one percentage point (used chiefly in expressing differences of interest rates).

96. India's first fast reactor fuel reprocessing plant to be built at which state?



A. Andhra Pradesh B. Gujarat

C. Tamil Nadu D. Chennai

E. Rajasthan

Answer (C) The foundation for India's first fast reactor fuel reprocessing plant to be built at a cost of Rs 9,600 crore is expected to be laid in two months' time at Kalpakkam in Tamil Nadu.

97. The 8th edition of Global Innovation Index (GII) 2015 was released jointly by Cornell University, INSEAD and the World Intellectual Property Organization (WIPO). What is the rank of India in the index?



A. 64 B. 76 C. 81

D. 95 E. 101

Answer (C) India's ranking has slipped further to 81 in the list, compared to 76 in 2014, but it remained at the top of the regional ranking of Central and Southern Asia. Switzerland, the United Kingdom, Sweden, the Netherlands and the United States of America are the world’s five most innovative nations.

98. Recently, G-20 group has launched the “Women-20 (W20)” group to empower women. The first Women-20 (W20)”summit 2015 will be held in ____.



A. Istanbul B. Ankara

C. Indonesia D. Beijing

E. Tokyo

Answer (A) To promote global gender-inclusive economic growth, the Group of Twenty (G20) launched Women-20 (W20) group. The first W20 summit 2015 will held on October 7-8, 2015 at Istanbul, Turkey. Ms. Gulden Turktan from Turkey has been appointed as the first President of W20 forum.

99. Reserve Bank of India (RBI) declared State Bank of India (SBI) and ICICI Bank as D – SIBs. What is the expand form as D – SIBs?



A. Domestic Security Internal Banks

B. Domestic Systemically Initial Banks

C. Domestic Systemically Important Banks

D. Demand Systemically Individual Banks

E. Deposit Systemically Interest Banks

Answer (C) The Reserve Bank of India (RBI) declared State Bank of India (SBI) and ICICI Bank as Domestic Systemically Important Banks (D- SIBs).

100. National Institution for Transforming India (NITI) Aayog has launched the open source web based tool by which name? A. IESS 2050 B. IESS 2047



C. IESS 2036 D. IESS 2022

E. IESS 2019

Answer (B) The second version of an open source web based tool, India Energy Security Scenarios 2047 calculator (IESS 2047) has been launched by the National Institution for Transforming India (NITI) Aayog.



101. According to the Socio Economic and Caste Census (SECC) which among the following states in India has highest per cent of Urban BPL households?



A. Bihar B. Uttar Pradesh

C. Manipur D. Jharkhand

E. Sikkim

Ans (C) The highest fraction of Urban Poors are in Manipur (54.95% of its total population), Mizoram (52.35% of its total population and Bihar (49.82 per cent). On the other hand, least proportion of Urban poor in India are in Goa (16%) and UTs of D&NH and Delhi (8% each of their population).

102. Union Government has decided to set up a Bank Board Bureau (BBB) to monitor key performance indicators of two dozen public sector banks (PSBs). BBB will start functioning from ___.



A. 31 December 2015 B. 1st January 2016

C. 31st March 2016 D. 1st April 2016

E. 1st June 2016

Ans (D) Bank Board Bureau (BBB) will start the functioning from next financial year i.e. from 1st April 2016.

103. Which among the following banks has launched country’s first MUDRA (Micro Units Development and Refinance Agency) card under the Pradhan Mantri MUDRA Yojana (PMMY)?



A. SBI B. PNB

C. Canara Bank D. Corporation Bank

E. Bank of Baroda

Ans (D) Corporation Bank launched country’s first MUDRA (Micro Units Development and Refinance Agency) card under the Pradhan Mantri MUDRA Yojana (PMMY) in Mangalore, Karnataka.

104. India and United States have signed Inter Governmental Agreement to implement the FATCA. This agreement will promote transparency on tax matters. Expand the term FATCA.



A. Financial Account Tax Compliance Act

B. Foreign Account Trade Company Act

C. Foreign Account Tax Compliance Act

D. Fiscal Account Tax Compliance Act

E. None of these

Ans (C) India and United States have signed Inter Governmental Agreement to implement the Foreign Account Tax Compliance Act (FATCA). This agreement will promote transparency on tax matters.

105. Which among the following statements are correct/s in the context with Payment Banks?



A. The minimum paid-up capital requirement for payments banks is Rs. 100 crore

B. Payment banks will offer both deposits as well as loan products.

C. Payments banks will have to invest in government securities with a maturity of up to 1 year

D. Payments banks can open small savings accounts and accept deposits of up to Rs.1 lakh per individual customer.

(1) A, C, D (2) B, C, D

(3) A, B, C (4) A, B, D

(5) All are true

Ans (1) A, C, D The main objective of payment banks is to increase financial inclusion in the country via a primary focus on domestic payments services by providing small savings accounts.

Note: Payments banks will be used only for transaction and deposits purposes. Unlike Small banks, payments banks cannot offer loan products to the customers.

106. Union Government has launched Indradhanush Mission to revamp functioning of public sector banks (PSBs). How many pronged plan has been classified in Indradhanush Mission?



A. Six B. Five

C. Seven D. Four

E. Eight

Ans (C) Union Government has launched a seven pronged plan called Indradhanush Mission to revamp functioning of public sector banks (PSBs). It was launched by Union Finance Minister Arun Jaitley in New Delhi.

The seven shades of Indradhanush mission include (i) appointments, (ii) Bank Board Bureau (BBB), (iii) capitalisation (iv) de-stressing PSBs, (v) empowerment, (vi) framework of accountability and (vii) governance reforms.

107. Recently, Asian Development lowered India’s Growth Forecast for 2016 from 7.8% to _____.



A. 7.6% B. 7.5%

C. 7.4% D. 7.3%

E. None of these

Answer (C) Asian Development Bank (ADB) had lowered India’s gross domestic (GDP) growth projections for the fiscal year (FY) 2016 to 7.4 percent from 7.8 percent projected in its March 2015 Outlook. For FY2015







7.8 percent which is lower than the earlier forecast of 8.2 percent.

108. The Employees Provident Fund Organisation (EPFO) increased the life insurance cover of its subscribers from 3.6 lakh to _____.



A. 6 lakh B. 5 lakh

C. 5.5 lakh D. 4 lakh

E. None of these

Answer (A) The Employees Provident Fund Organisation (EPFO) on 16 September 2015 increased the life insurance cover of its subscribers from 3.6 lakh to 6 lakh rupees. About 40 million contributing members of the EPFO will be benefitted from the decision taken at the 208th CBT meeting in Hyderabad.

109. What is the full form of CVV?



A. Credit Verification Value

B. Currency Verification Variable

C. Customer Verification Value

D. Card Verification Value

E. None of these

Answer (D) Full form of CVV is - "Card Verification Value" on your credit card or debit card is a 3 digit number on VISA, MasterCard etc.

110. The selling of life assurance and other insurance products and services by banking institution is known as ____.



A. Insurance cover B. Bancassurance

C. Protection cover D.Assurance cover

E. None of these

Answer (B) The selling of life assurance and other insurance products and services by banking institution is known as Bancassurance.

111. In what denominations Commercial Paper (CP) can be issued?



A. Rs. 1 lakh B. Rs. 2 lakh

C. Rs. 5 lakh D. Rs. 10 lakh

E. None of these

Answer (C) Commercial Paper (CP) can be issued in denominations of Rs.5 lakh or multiples thereof. Commercial Paper (CP) is an unsecured money market instrument issued in the form of a promissory note.

112. What is the loan limit for education under priority sector for studies abroad?



A. Rs. 30 lakh B. Rs. 25 lakh

C. Rs. 20 lakh D. Rs. 15 lakh

E. Rs. 10 lakh

Answer (C) Loans to individuals for educational purposes including vocational courses upto Rs.10 lakh for studies in India and Rs. 20 lakh for studies abroad are included under priority sector.

113. What is the time limit for an asset or loan to be declared as Non-Performing Asset?



A. 30 days B. 60 days

C. 90 days D. 120 days

E. 150 days

Answer (C) A Non-performing asset (NPA) is defined as a credit facility in respect of which the interest and/or installment of Bond finance principal has remained ‘past due’ for a specified period of time. Once the borrower has failed to make interest or principle payments for 90 days the loan is considered to be a non-performing asset

114. Reserve Bank will issue _____bank notes with 3 additional features to aid the visually impaired in identification.



A. Rs 500 and Rs 1,000 B. Rs. 100 and Rs. 500

C. Rs. 50 and Rs. 100 D. Rs. 10 and Rs. 100

E. None of these

Answer (A) Reserve Bank will issue Rs 500 and Rs 1,000 bank notes with 3 additional features to aid the visually impaired in identification. The central bank said the new Rs 1,000 and Rs 500 denomination bank notes will have numerals in both the numbering panels in ascending size from left to right, while the first three alpha-numeric characters will remain constant in size.

Further, Rs 500 denomination bank notes will have five angular bleed lines in three sets of 2-1-2 lines on the obverse in both, the upper left and the right hand edge of the bank note. While, Rs 1,000 bank notes will have six angular bleed lines in four sets of 1-2-2-1 lines on the obverse in both, the upper left and right hand edge of the bank notes.

115. Recently, India has purchased Boeing's Apache and Chinook helicopters in a deal of worth around $2.5 billion from which country?



A. Russia B. France

C. US D. UK

E. None of these

Answer (C) India has purchased Boeing's Apache and Chinook helicopters in a deal of worth around $2.5 billion from United States.

116. Foreign exchange Reserves in India are kept in the custody of ___.



A. Govt. of India B. Finance Ministry

C. Reserve Bank of India D. Overseas Ministry

E. SEBI

Answer (C) Foreign exchange Reserves in India are kept in the custody of Reserve Bank of India.





117. Who is heading the committee to evaluate applications received for the proposed small finance banks that was constituted by the RBI in 2014?



(A) Usha Thorat (B) Shyamala Gopinath

(C) Deepak Parekh (D) Bimal Jalan

(E) None of these

Ans: (A) Usha Thorat is the head of the committee to evaluate applications received for the proposed small finance banks in India.

118. Simply click a card launched by SBI special for ____.






A. Cash Transfer B. Online Shopping

C. Pay Utility Bills D. For Home Loans

E. None of these

Answer (B) Simply click a card launched by SBI special for Online Shopping.

119. A receipt listed in India and traded in rupees declaring ownership of shares of a foreign company:



A. Indian Depository Receipt (IDR)

B. Commercial Paper

C. Promissory Note

D. Indian Depository Revenue

E. None of these

Answer (A) A receipt listed in India and traded in rupees declaring ownership of shares of a foreign company is known as Indian Depository Receipt (IDR)

120. Recently, RBI declared which of the following as Domestic Systemically Important Banks (D- SIBs)?



A. SBI & HDFC B. SBI & ICICI

C. ICICI & HDFC D. SBI & BOB

E. SBI & PNB

Ans (B) The Reserve Bank of India (RBI) recently, designated State Bank of India (SBI) and ICICI Bank Ltd, the country’s two largest lenders, as Domestic Systemically Important Banks (D-SIBs). The banks have been named Domestic Systemically Important Banks (D-SIBs), with SBI falling in bucket three while ICICI Bank is in bucket one.





Important Banking Awareness Questions

From Previous year Bank Exams





1. MICR code consists of how many digits?



Ans: 9 digits.

(First three digits denotes city, next three digits representing the bank and the last three digits representing the bank branch)

2. What is the minimum limit in RTGS system?



Ans: 2 lakhs (there is no upper limit in RTGS)

3. What is full form of CTS?



Ans: Cheque Truncation System

4. Under which service, customers may access their bank account and perform basic transactions from any of the member branch offices.



Ans: Core Banking Solution (CBS)

5. Exchange of cash flow in different currency is known as:



Ans: Currency Swap

6. Assets or loans which stop performing after 90 days is known as:



Ans: Non Performing Asset (NPA)

7. Who controls the Monetary Policy in India?



Ans: RBI (Reserve Bank of India)

8. Which card is issued by NPCI (National Payments Corporation of India)?



Ans: RuPay Card

9. Definition of Current Account deficit:



Ans: A measurement of a country's trade in which the value of goods and services it imports exceeds the value of goods and services it exports.

10. Full form IFSC –



Ans: Indian Financial System Code

11. Commercial paper can be issued for a maximum period of:



Ans: 365 days or 1 year.

12. The Mutual funds in India follow accounting standards laid by:



Ans: SEBI (Securities and Exchange Board of India)

13. Minimum amount for Certificate of Deposit has been fixed at:



Ans: Rs. 1 Lakh

14. AML is a term mainly used in the financial and legal industries. Expand the term AML:



Ans: Anti Money Laundering

15. PIN is a number allocated to an individual and used to validate electronic transactions. Expand PIN:



Ans: Personal Identification Number

16. What is Repo rate?



Ans: It is the rate is the rate at which RBI lends money to the commercial banks.



17. What is Stale Cheque?



Ans: A cheque which is presented to a bank after 3 months from date of issue is considered as stale cheque and will often not be honored for cash or deposit at a bank.

18. What is Bancassurance?



Ans: The selling of life assurance and other insurance products and services by banking institutions.

19. The objective of KYC guidelines is to prevent banks from being used, intentionally or unintentionally, by criminal elements for money laundering or terrorist financing activities. What is the full form of KYC?



Ans: Know Your Customer (KYC)

20. Know Your Customer (KYC) guidelines are issued under:



Ans: Section 35A of the Banking Regulation Act, 1949

21. In BSBDA (Basic Savings Bank Deposit Account) the credits in a financial year does not exceed rupees:



Ans: Rs. 1 lakh

22. In BSBDA (Basic Savings Bank Deposit Account) the balance at any point of time does not exceed rupees:



Ans: Rs. 50,000

23. In BSBDA (Basic Savings Bank Deposit Account) the withdrawals and transfers in a month does not exceed rupees:



Ans: Rs. 10,000

24. At which rate RBI give loans to commercial banks?



Ans: Repo rate

25. Full form of CASA:



Ans: Current Account Saving Account

26. In what denominations Commercial Paper (CP) can be issued?



Ans: Rs. 5 lakh

27. What is the minimum denomination of Treasury bills to issue in India?



Ans: Rs. 25,000

28. Who cannot issue Certificate of Deposit (CD)?



Ans: Regional Rural Banks (RRBs) and Local Area Banks (LABs)

29. Expand ASBA:



Ans: Application Supported by Blocked Amount

30. Depositor Education and Awareness Fund (DEAF) is maintained with:



Ans: RBI

31. Cash Reserve Ratio (CRR) is the amount of funds that the banks have to keep with:



Ans: Central Bank (RBI)

32. What is the maximum amount per transaction NEFT limit for cash-based remittances to Nepal?



Ans: Rs. 50,000

33. What does CAR stands for?



Ans: Capital Adequacy Ratio

34. IFSC code consists of _____ alpha numeric code.



Ans: 11 digits

(The IFSC is an 11 digit alpha numeric code, with the first four digits identifying the bank, fifth is numeric (kept 0) and the last six digits represent the bank branch.)

35. When money is lent or borrowed for one day or on overnight basis it is known as:



Ans: Call Money

36. When money is lent or borrowed for between 2 days and 14 days it is known as:



Ans: Notice money

37. When money is lent or borrowed for a period of more than 14 days, it is known as:



Ans: Term money

38. Treasury Bills and Certificate of Deposit are considered as the ____instruments.



Ans: negotiable money market

39. Commercial Paper (CP) is an unsecured money market instrument issued in the form of a _____.



Ans: Promissory note.

40. What is the upper limit in Public Provident Fund (PPF)?



Ans: Rs. 1.5 lakhs

41. As per RBI guidelines, with effect from April 1, 2012, the validity period of Cheques, Demand Drafts, Pay Orders and Banker's Cheques is _____.



Ans: 3 months

42. What is the time limit for an asset or loan to be declared as Non-Performing Asset?



Ans: 90 days

43. Deposit Insurance and Credit Guarantee Corporation (DICGC) does not cover:



Ans: Primary co-operative societies

44. RBI measure to liquidate the market:



Ans: Repo rate.

45. Full form of EFT:



Ans: Electronic Fund Transfer

46. Fastest mode of transaction:



Ans: RTGS

47. Alphanumeric code on cheque is known as:



Ans: Indian Financial System Code (IFSC)

48. Bank pays interest on savings account?



Ans: Daily Basis





49. What is the loan limit for education under priority sector for studies abroad?



Ans: Rs. 20 lakh

(Loans to individuals for educational purposes including vocational courses upto Rs.10 lakh for studies in India and Rs. 20 lakh for studies abroad are included under priority sector.)

50. RuPay Card is an Indian version of credit/debit card is launched by which organization:



Ans: NPCI (National Payments Corporation of India)

51. Fixed Deposit (FD)Account may be opened for a minimum period of:



Ans: 7 days

52. What is the minimum amount required to open a Fixed Deposit (FD)?



Ans: Rs.1000

53. The Banks has converted all ‘no - frills' accounts’ into:



Ans: Basic Savings Bank Deposit Accounts

54. 'Pradhan Mantri Jan Dhan Yojana' is a Scheme for:



Ans: Financial inclusion

55. How much overdraft facility to be provided in 'Pradhan Mantri Jan Dhan Yojana' scheme?



Ans: Rs. 5,000

56. In CBS, C stands for:



Ans: Core

57. In CRAR, A stands for:



Ans: Assets (Capital to Risk Weighted Assets Ratio)

58. IMPS - Immediate Payment Service is an interbank electronic instant mobile money transfer service through mobile phones in India, the facility is provided by:



Ans: NPCI (National Payment Corporation of India)

59. The Central Bank of India has adopted new measure of inflation:



Ans: Consumer Price Index (CPI)

60. When a cheque is torn into two or more pieces and presented for payment, such a cheque is called:



Ans: mutilated cheque

61. How much fee charged to file a complaint under Banking Ombudsman?



Ans: Banking Ombudsman does not charge any fee

62. Who is the appellate authority in Banking Ombusdsman?



Ans Deputy Governor of RBI

63. If any customer is not satisfied by the decision of Banking Ombudsman, customer can appeal against the award before the appellate authority within how many days from the date of receipt?



Ans 30 days

64. ATMs or Cash Dispensing machine which are owned and operated by Non-Banking Financial Companies are called:



Ans: White Label ATMs.

65. RBI gave in-principle for Banking license to:



Ans: IDFC and Bandhan

66. Minimum capital requirement for new banks in private sector is:



Ans: Rs. 500 crore

67. A Non-Banking Financial Company (NBFC) is a company registered under the:



Ans: Companies Act, 1956

68. Minimum capital requirement for Non-Banking Financial Company (NBFC) is:



Ans: Rs. 500 crore

69. The NBFCs are allowed to accept/renew public deposits for a minimum period of:



Ans: 12 months ( and maximum period of 60 months)

70. A NBFCs cannot offer interest rates higher than the ceiling rate prescribed by RBI? What is the present ceiling?



Ans: 12.5 per cent per annum

71. Minimum capital requirement for Foreign banks that want to set up operations in India is:



Ans: Rs. 500 crore

72. What is the minimum paid-up capital requirement of both small banks and payments banks in India?



Ans: Rs. 100 crore

73. RBI extended the timeline for full implementation of Basel III norms till:



Ans: 31 March 2019

74. SLR ((Statutory Liquidity Ratio)is the amount a commercial banks needs to maintain in the form of _______before providing credit to its customers.



Ans: cash, or gold, or govt. approved securities (Bonds)

75. _______ is the rate at which banks borrow funds overnight from the Reserve Bank of India (RBI) against approved government securities.



Ans: Marginal Standing Facility (MSF)

76. Who issues Treasury bills (T-bills) in India?



Ans: Government of India

77. Treasury bills are available for a minimum amount of:



Ans: Rs. 25,000

78. Minimum & Maximum Limit of NEFT:



Ans: no limit

79. Full form of CASA –



Ans: Current Account Saving Account





80. Govt. to implement GST (Goods & Services Tax) new indirect tax regime from:



Ans: 1st April 2016

81. RBI extends deadline for exchanging pre-2005 currency from June 30, 2015 to



Ans: December 31, 2015

82. How much amount RBI allows in 'Tap and pay' transactions without PIN?



Ans: Rs. 2000

83. RBI lifted a ban on carrying Indian bank notes of Rs 1,000 and Rs. 500 denominations to and from:



Ans: Nepal and Bhutan

84. C stands for in "KYC":



Ans: Customer

85. Pension scheme for unorganized sector:



Ans: Swavalamban

86. Full form of REIT:



Ans: Real State Investment Trust

87. Forward Market Commission merge with:



Ans: SEBI

88. Purpose of starting Minor account for 10 years children:



Ans: Financial inclusion

89. Full form of CVV:



Ans: Card Verification Value

90. Bank of International Settlement headquarter:



Ans: Switzerland

91. Who appoints Banking Ombudsman officer?



Ans: RBI

92. IFRS full form:



Ans: International Financial Reporting Standards.

93. Corporate Social Responsibility committee headed is by Ans: Anil Baijal

94. Which is the Regulatory body for RRBs?



Ans: NABARD

95. RBI policy related to money laundering?



Ans: Know Your Customer

96. Mutual funds regulated by:



Ans: SEBI (Securities and Exchange Board of India)

97. What is the full form of MTSS?



Ans: Money Transfer Service scheme

98. How much amount of money can RBI lend to a bank?



Ans: 2% of NDTL

99. What RBI does to Increase its Monetary Base?



Ans: OMO Open Market Operations

100. Which rate does RBI Reduces to Increase liquidity in market?



Ans: Repo Rate

101. After completion of 15 years, Public Provident Fund (PPF) can be extended up to how many years?



Ans: 5 years

102. Short term Money lending process is known as:



Ans: Call Money

103. Treasury bill tenure:



Ans: 91 days, 182 days, 364 days

104. Minimum limit for medium scale enterprises is Rs. 5 Cr. what is maximum limit?



Ans: 10 Cr.

105. Prime Lending Rate is replaced by:



Ans: Base rate

106. Banks cannot grant Loan below which rate:



Ans: Base Rate

107. If RBI reduces CRR, what happens:



Ans: Credit Supply increases, loans get cheaper.

108. Name the scheme to include every people under banking system:



Ans: Financial Inclusion

109. Special Drawing Right (SDR) is a monetary unit of:



Ans: IMF

110. Cheque which is not crossed is called:



Ans: Open cheque

111. Teaser rates are related to which type of loans:



Ans: Home loans

112. What is Teaser loan?



Ans: If a bank offers a slightly lower rate in the initial years and higher rate in later years, it is called a teaser loan.

113. The RBI policy rate which is purely an indicative rate used by RBI to signal long – term outlook on interest rates is:



Ans: Bank rate

114. The term ‘pre – shipment’ finance relates to:



Ans: export credit

115. A receipt listed in India and traded in rupees declaring ownership of shares of a foreign company:



Ans: Indian Depository Receipt (IDR)

116. With effect from July 2012, for calculating of lending rates, the RBI has advised banks to switch over to the:



Ans: Base Rate systems

117. Mobile banking fund transfer limit in a day:



Ans: Rs. 50,000

118. The seed capital of Bhartiya Mahila Bank is:



Ans: Rs.1000 crore

119. “Lender of the Last Resort” by Banks is known as:



Ans: RBI

120. “Fixed deposit” is also referred to as:



Ans: Term Deposit





121. The holidays for the banks are declared as per:



Ans: Negotiable Instruments Act

122. In banking business, when the borrowers avail a term Loan, initially they are given a repayment holiday and this is referred as:



Ans: Moratorium

123. Regulator of Micro, Small and Medium enterprises in India:



Ans: SIDBI (Small Industries Development Bank of India)

124. A worldwide financial messaging network which exchanges between banks and financial institutions is known as:



Ans: Structured Financial Messaging System (SFMS)

125. The term “Smart Money” refers to:



Ans: Credits Card

126. The maximum deposit amount insured by DICGC?



Ans: Rs. 1 lakh per depositors across all banks

127. With reference to a cheque which is the ‘drawee bank’?



Ans: The bank upon which the cheque is drawn

128. In which of the following fund transfer mechanisms, can funds be moved from one bank to another and where the transaction is settled instantly without being bunched with any other transaction?



Ans: RTGS

129. Bad advances of a Bank are called:



Ans: Non – performing Asset

130. By increasing repo rate, the economy may observe the following effects:



Ans: rate of interest on loans and advances will be costlier

131. Increased interest rates, as is existing in the economy at present will:



Ans: mean higher cost of raw materials

132. The sole authority to issue and manage currency in India:



Ans: RBI

133. In India, one- rupee coins are issued by:



Ans: Govt. of India

134. Fixed deposits and recurring deposits are:



Ans: repayable after an agreed period

135. When a bank returns a cheque unpaid, it is called:



Ans: dishonour of the cheque

136. What is ‘Demat Accounts’?



Ans: Accounts in which shares of various companies are traded in electronic form

137. When the rate of inflation increases:



Ans: purchasing power of money decreases

138. Banks in India are regulated under:



Ans: Banking Regulation Act, 1949

139. Banking sector falls under which of the following sectors?



Ans: Service Sector

140. ASBA scheme is related to the purchase of:



Ans: IPO

141. In a bank, which of the following are the usual types of accounts?



Ans: Current accounts, Savings bank accounts and Term deposit accounts

142. The most powerful tool used by the Reserve Bank of India to control inflation is to:



Ans: raise interest rates

143. NEFT and RTGS in banking terminology speak of:



Ans: electronic fund transfer from bank to bank

144. The ownership of public sector banks rests:



Ans: jointly with the Government of India and the shareholders from the public

145. If a cheque is postdated, the bank on which it is drawn:



Ans: will not honour the cheque before the date of the cheque

146. Regulator of Capital Market in India:



Ans: SEBI

147. First Indian Bank to introduce credit card:



Ans: Central Bank of India

148. RBI nationalized in:



Ans: January 1, 1949

149. Loans of very small amounts given to low income groups is called:



Ans: Micro Credit

150. RBI established in:



As: April1, 1935

No comments:

Post a Comment